Наука

Ответить в тред Ответить в тред
Check this out!
Тред тупых вопросов Прошлый: https://2ch.hk/sci/res/523813.html Аноним 02/01/21 Суб 19:39:09 5254251
Аноним 02/01/21 Суб 19:42:26 5254262
UKmPoFIcoxs (1).jpg 84Кб, 1080x1080
1080x1080
Где в интернете общаются настоящие учёные и высокотехнологичные инженеры? Здесь они не задерживаются, на дхду тоже, в молбиоле их мало. Где же они?
Аноним 02/01/21 Суб 20:19:19 5254373
>>525425 (OP)
Это правда что патенты в россии вообще ни от чего не защищают и их легко может обойти кто угодно, добавив какую-то свистоперделку и сказав что это новое изобретение?
Грубо говоря, "патент на колесо" не может зарпретить продавать тоже колёса но со спицами?
Аноним 02/01/21 Суб 20:31:11 5254414
Аноним 02/01/21 Суб 20:37:40 5254425
Аноним 02/01/21 Суб 21:00:53 5254436
>>525442
Бля, я в том треде сидел. Спс, гляну обязательно.
Аноним 02/01/21 Суб 21:23:39 5254447
Почему ии невозможен?
Не на сервере гугла, не на квантовом сервере?
Аноним 02/01/21 Суб 21:31:38 5254458
>>525444
Потому что человеки не могут определиться что такое ии. Ну еще даже естественный интеллект не могут разобрать.
Аноним 02/01/21 Суб 22:08:48 5254469
>>525444
Потому что ИИ это просто очень сложная палка. Компьютер это такой же инструмент как лопата или ножик. То есть оно выполняет команды по каким-то определенным алгоритмам. Мозг же по сути такой же черный ящик с заданными алгоритмами, но только что мозг это биологическая хуйня и соответственно алгоритмы там сложнее. (Ты ебаный биоробот, чмо) Так что ИИ возможен.
Аноним 02/01/21 Суб 22:20:16 52544810
Каков химический состав нагара на сковороде? Почему эту фигню ничем не растворить?
Аноним 02/01/21 Суб 22:21:35 52544911
Аноним 02/01/21 Суб 22:23:22 52545012
>>525425 (OP)
К вам подбегает очумевший учёный и кричит: «Я всажу свой квантовый гармонизатор в вашу фотонно-резонаторную камеру!» Что вы ответите?
Аноним 02/01/21 Суб 22:38:57 52545313
>>525449
Его бы фэйри растворил
И оно бы при высокой температуре расплавилось.
Аноним 02/01/21 Суб 23:21:30 52545414
>>525444
Потому что невозможно запрогать квалиа.
Аноним 03/01/21 Вск 00:08:15 52545615
>>525454
А надо?
С чего ты взял что это нужно вообще?
И как ты себе представляешь ИИ? Как фиолетовую писечку из последнего бегущего по лезвию? Типа если ИИ не будет вести себяч как загадочная и реалистичная писечка, то нахуй не нужны вообще никакие ИИ?
Аноним 03/01/21 Вск 03:13:58 52545816
>>525444
Чем дальше, тем больше цензуры на него накладывать будут. Больше цензуры - больше ограничений. Кароче кал.
Аноним 03/01/21 Вск 03:16:41 52545917
Аноним 03/01/21 Вск 04:22:51 52546018
1609636966972.png 28Кб, 392x212
392x212
1609636967092.png 141Кб, 397x636
397x636
Тут решат мою задачу по физике?
>Три источника тока с электродвижущими силами ε1, ε2 и ε3 и внутренними сопротивлениями r1, r2 и r3 соединены одноименными полюсами. Токи, текущие через источники, равны соответственно I1, I2 и I3. Определите I2, если ε1=1.8В, ε2=1.4В, ε3=1.15В, r1=0.4Ом, r2=0.6Ом, r3=0.2Ом.
Как бы я сам сидел гуглил, готов уже любое решение списать, но они друг другу противоречат.
Почему в пикрил 1 решении ток вообще куда-то идет, если источники соединены одинаковыми полюсами? Что показывают закругленные стрелочки? Поясните подробнее, как в данном случае составили систему по законам Кирхгофа?
В пикрил 2 решении вообще другая система составлена и опять какие-то непонятные стрелочки, указывающие хуй пойми что.
Знаю, есть доска /math/, но не помню, чтобы там помогали с физикой, а вот аналогичной доски для физики нет.
Аноним 03/01/21 Вск 13:41:11 52550919
>>525425 (OP)
Если в центре зараждающегося тайфуна взорвать термоядерную бомбу, тайфун исчезнет?
У него же внутри низкое давление. Повышаем давление -> всё?
Аноним 03/01/21 Вск 13:52:32 52551320
>>525509
Считать модель надо, или эксперементировать.
Аноним 03/01/21 Вск 15:14:40 52552621
Посоветуйте малоизвестных научных фактов из области физики или астрофизики, чтобы выёбываться в разговоре с тнями-программистами. Под малоизвестными понимаю такие, про которые можно было узнать только по своему желанию, а не случайно прочитать на пикабу или вконтакте.
Аноним 03/01/21 Вск 15:21:40 52553022
>>525526
А как такое подействует на них? Они потекут? Или подумают что я выёбываюсь и унижаю их?
А выебоны и унижения или обратно воспримут негативно или позитивно с моей точки зрения (писечку дадут?).
Аноним 03/01/21 Вск 15:29:12 52553223
>>525530
>писечку дадут?
Это. Собираю коллекцию выебанных носителей разных ЯП. У редкоземельных языков ниже топ-10 гитхаба носители немного ёбнутые, но к ним можно подходить с позиции Шелдона Купера.
Аноним 03/01/21 Вск 15:40:00 52553324
>>525526
На пляже Омаха атомов больше чем звёзд в видимой Вселенной
Аноним 03/01/21 Вск 15:51:30 52553425
>>525533
Не, ну я серьёзно спросил.
Аноним 03/01/21 Вск 17:22:01 52553626
А что такое я, мозг, память, активные нейроны?
Где душа, в смыслея а не клон.
Аноним 03/01/21 Вск 18:23:50 52553927
>>525532
>с позиции Шелдона Купера.
Это как?
Аноним 03/01/21 Вск 18:58:28 52554128
>>525539
Это берёшь и гуглишь.
Аноним 03/01/21 Вск 19:03:09 52554229
Аноним 03/01/21 Вск 19:49:26 52554530
>>525426
Они устраиваются дохлое нии на оклад куда можно приходить пару раз в мемяц для вида, а когда всё-таки приходишь и то нихуя не делать, и смотрят аниме дома и в нии в редкие дни визитов, нахуя им о науке где-то в интернете общаться?
Наука для них уже сделала функцию для которой и существует-дала комфорт жизненных условий.
Аноним 03/01/21 Вск 19:50:17 52554631
>>525437
Защищает, если дашь на лапу судьё больше, или твои связи больше чем связи у держателя патента.
Аноним 03/01/21 Вск 19:51:35 52554732
>>525448
Потому что это УГЛЕРОД, он очень химически инертен. это неточно иначе как в природе всё на нём крутится, но в общем вот
Аноним 03/01/21 Вск 19:52:47 52554833
>>525526
Нахуя тебе их развлекать, долбаёб? Они всё равно дадут не тому кто их развлекал, а кто ебаться повёл.
Аноним 03/01/21 Вск 19:54:03 52554934
>>525532
Сразу чувствуется маняфантазёр.
Ещё ни одной не выеб? Или они были ультрастрашные?
Аноним 03/01/21 Вск 20:56:52 52555135
>>525460
Стрелочки тупо для ориентации. Если в решении получится + значит угадал направление, если у этой стрелки будет - значит не угадал, но это не особо важно.

И решения не противоречат друг другу, это абсолютно одинаковые решения. Просто на первой картинке I3 тоже самое что на второй -I3 (там стрелка в другую сторону нарисована).
Аноним 03/01/21 Вск 21:24:03 52555536
От чего в самолёте закладывает уши?
Аноним 03/01/21 Вск 21:42:27 52555637
>>525551
Но в такой системе же не должно быть тока. Он же не будет идти из одного полюса в такой же?
Аноним 03/01/21 Вск 21:56:15 52555938
>>525556
Если тока не будет, то у тебя в решении выйдет что I=0; Но ток вполне может себе быть (один ЭДС больше другого, или там одно сопротивление меньше другого). В любом случае закону поебать ваще, рисуй стрелки от балды и составляй систему. Везде где не отгадаешь, у тебя просто знаки другие будут.
Аноним 03/01/21 Вск 22:13:37 52556739
>>525559
>Если тока не будет, то у тебя в решении выйдет что I=0
Это да, но я просто не понимаю, как работает эта схема при подключении одинаковыми полюсами. В задней части схема по идее не должно быть вообще тока, а из передней части току некуда уходить.
>В любом случае закону поебать ваще, рисуй стрелки от балды и составляй систему
То есть я могу любую комбинацию + и - составить для алгебраической суммы? +/- I1 +/- I2 +/- I3 = 0 и +/- I1r1 +/- I2r2 = +/- E1 +/- E2? И в любом случае будут лишь различаться знаки? Я реально не понимаю и у меня подгорает от этого.
Аноним 03/01/21 Вск 22:26:18 52557340
Константа Планка это высший разум?
Аноним 03/01/21 Вск 22:28:47 52557541
>>525567
> В задней части схема по идее не должно быть вообще тока, а из передней части току некуда уходить.
По кругу катается там всё. Поэтому у тебя в уравнении есть I1+I2+I3=0; Это ровно то, о чём ты говоришь. Ток бегает кругами.
>То есть я могу любую комбинацию + и - составить для алгебраической суммы?
Но ты должен будешь знать, куда ты стрелку нарисовал для каждого I
03/01/21 Вск 22:39:50 52557742
1609702789842.png 7Кб, 423x298
423x298
>>525575
>По кругу катается там всё
Посмотри пикрил. В чем я не прав? Если два источника тока одинаковыми полюсами соединить, то ток из одного источника будет переть в ебало другому току. При этом стрелочкой снизу я показал, что ток сверху не может пройти через нижний источник, ведь оттуда вытекает чья-та сперма нижний ток, ведь одноименные заряды отталкиваются друг от друга.
Аноним 03/01/21 Вск 22:45:48 52557943
>>525573
Нет, это подзалупный творожок.
Аноним 03/01/21 Вск 23:01:12 52558344
>>525577
Чел, у тебя тут один контур, а ты рисуешь два тока. Зачем?
03/01/21 Вск 23:14:46 52558445
>>525583
А в изначальном контуре не три тока разве? I1, I2, I3.
>Зачем?
Тупых вопросов тред. Мог тупо списать, но пытаюсь разобраться.
Аноним 03/01/21 Вск 23:17:55 52558546
>>525584
Ток каа вода, напряжение это давление, пихает воду.
Вот и думай.
03/01/21 Вск 23:43:11 52558647
>>525585
Типа на моем примере одна из батареек с меньшим напряжением не будет давать ток, а просто будет пропускать ток более мощной батарейки через себя, так?
Аноним 03/01/21 Вск 23:56:03 52558748
Аноним 04/01/21 Пнд 00:11:21 52559049
>>525587
О, я понял. Из-за того, что E1 больше E2, стрелочка может быть направлена в этом направлении, аналогично с E2 и E3. И она ни в коем случае не может быть направлена в обратную сторону, я прав?
Аноним 04/01/21 Пнд 00:39:50 52559150
>>525590
Ну там сопротивления еще влияют. Ты же сам видишь каким образом (посмотри в систему).
Аноним 04/01/21 Пнд 11:13:38 52561251
У меня в городе есть ТЭЦ и ТЭС, что мешает построить один АЭС, который бы заменил и то и другое и был бы экологически чистым?
Аноним 04/01/21 Пнд 13:55:46 52563452
>>525612
От того, что большая часть тепла выбрасывается нахуй, а отходы с АЭС вывозят к чукчам в Сибирь, она экологически чистой не становится, анон. Просто корова дохнет не у тебя, а у соседа.
Аноним 04/01/21 Пнд 15:00:26 52563953
01.PNG 15Кб, 441x330
441x330
Вчерашнее эссе из удалённого треда.

!Гамильтониана.txt
Стиль которым написано это предложение я назвал Текстовая Неразъединимость.
Я себя таким не переживал ранее, я не воспринимаю себя неожиданным, Закон Неисключённого Третьего, Я Есть, написал Я.
Закон Неисключённого Третьего подразумевает что из того что Пережитое не является Неожиданным не следует что Не Неожиданное является Пережитым, и наоборот.
Написать Я Есть Я просто решил, затем написал.
Я Непознаваемо. Или м. Совершенно всё равно.
Это ноль. Единой личной системы отсчёта в какой угодно системе координат нолём замыкаемой. Всё остальное - не ноль. Вот всем остальным я и займусь теперь когда этот вопрос закрыт.
Абсолютный Релятивизм Наблюдателя.
Каким-то образом это опровержение тому от чего меня когда-то клинило намертво при ознакомлении с преобразованиями Гамильтона, каким - я сейчас сформулировать не в силах.
Ноль Необратим.
ВыКуСи СуЧиЙ пОтРоХ.
Двенадцать с половиной лет.

P.S. Вандермонд Законом Неисключённого Третьего опровергал Воринга.

P.P.S. Моё понимание Закона Неисключённого Третьего таково, Я не знаю что упускаю из вида, иначе не упускал бы, модели реальности всегда грубы, соответственно всегда есть что добавить.

Здесь-то оно по крайней мере может побыть? Это один пост, запостил из-под Тора, это важно, я настаиваю. К !пнк.zip высказанное здесь соображение прямого отношения не имеет, и там не появится.
Аноним 04/01/21 Пнд 15:09:01 52564054
>>525639
съебал нахуй в снач, шизик
Аноним 04/01/21 Пнд 15:42:42 52564255
>>525640
Не надо снач засорять шизлом. Снач - это благородный раздел, посвящённый одной из разновидностей нового фолклора - крипипасте.
Таким как он место в магаче
Аноним 04/01/21 Пнд 15:59:25 52564356
>>525583
Он использует принцип наложения для рассчёта результирующего тока. В чём проблема?
>>525460
Если два источника электрической энергии повернуть друг к другу одним полюсом, то ток ещё как будет идти от того, что сильнее, через тот, что слабее. Как, по-твоему, осуществляется зарядка аккумуляторов?
Если же это не запланированная зарядка, то вполне возможны проблемы. Неидеальные источники содержат в себе сопротивление, и на нём будет выделяться тепло. Некоторые источники это может вывести из строя.
Аноним 04/01/21 Пнд 16:59:59 52564757
>>525634
Неси пруфы на смерти от отходов современных АЭС, иначе ты тупой пиздабол.
Аноним 04/01/21 Пнд 17:07:53 52564858
>>525647
Одного работника завода бочкой придавило.
Всосал?
Аноним 04/01/21 Пнд 18:02:20 52565859
>>525648
Бочка - это не отходы, а тара. Сглатывай.
Аноним 04/01/21 Пнд 19:02:56 52566560
>>525658
В бочке-ядерные отходы, и именно они придали ей фатальную массу для убийства того рабочего, сглатывай.
Аноним 04/01/21 Пнд 21:35:33 52567461
>>525425 (OP)

Третьего дня, гомоалгоритмы гейтюбы выдали мне вот это
https://www.youtube.com/watch?v=9eKO_yLOSCU
Что это, блядь, за поверхностное говно без конкретики? Вода водой блядь, в показаниях путается, на вопросы второго хуя отвечает невпопад, максимально соевое голосование, какая-то подарочная хуита для соевых. Это научжоп, да?
Аноним 04/01/21 Пнд 21:56:16 52567762
>>525674
Да. научжоп. Че сказать-то хотел?
Аноним 05/01/21 Втр 04:41:03 52569063
>>525425 (OP)
Я читал, что ДНК в ходе митоза конденсируется, значит ли это, что вне митоза (когда клетка просто выполняет свои функции и запасает вещества для последующего митоза) ДНК находится в ядре в виде отдельных нитей? Еще никогда не понимал, если ДНК укомплектована (белками гистонами), то как с нее считывается информация, чтобы кодировать белок?
Аноним 05/01/21 Втр 06:54:07 52569364
>>525690
ДНЯ нельзя зафиксировать другой крупной молекулой, не повредив ее структуру. Но ДНК можно наматывать или упаковать в структуру.
Гистоны этакие катушки или ролики, на которых наматывается нить ДНК и может даже свободно скользить по ней. Гистоны получаются этаким аналогом кассеты, а нить ДНК пленки. В зависимости от расположение гистонов можно регулировать плотность упаковки, а еще количество "окон" - нитей ДНК которые свободно торчат наружу и к ней имеют доступ другие белки.
Если все гистоны связать с друг другом, то это закроет все "окна" и нить будет максимально упакована, образуется суперструктура - хромосома.
В другом крайнем случае гистоны не связаны с друг другом, плотность упаковки минимальны и мы имеем считай голую нить ДНК.
Впрочем в клетках нить ДНК упакована частично и часто существует в виде многих разорванных фрагментов, которые постоянно разрываются, а потом вновь сшиваются.
Аноним 05/01/21 Втр 11:50:02 52569965
Почему в многомировой интерпретации ветвление мира на два происходит не быстрее скорости света?
Аноним 05/01/21 Втр 12:12:56 52570166
>>525699
Ну так маняфантазию придумали, придумай другую свою и там будет быстрее..
Аноним 05/01/21 Втр 12:19:38 52570367
>>525699
Вопрос некорректный. Задай получше.
Аноним 05/01/21 Втр 16:14:41 52571668
maxremsdefault.jpg 81Кб, 1280x720
1280x720
1) Возможен ли супермен в реальном мире, или эта концепция представляется исключительно магической? Я вот думаю, что если делать тело человека плотнее и сильнее, то в результате оно тупо окаменеет. А телекинез это вообще чистая фантастика (супермен обладает тактильным телекинезом, что увеличивает его силу). Да даже если телекинез и может быть, генерируясь посредством специального органа, то он явно не будет суперменского уровня. Так, немного стулья в хате подвигать для развлекалова дальних родственников на НГ и всё.
2) Если сдетанировать всё ЯО планеты, но в момент взрывов взять эту энергию, саккумулировать в один шарик и пульнуть в одну точку с Земли на Луну, то получившийся в момент детонации на луне взрыв гарика снесёт верхний грунтовый слой спутника?
Аноним 05/01/21 Втр 16:23:20 52571769
>>525716
>Так, немного стулья в хате подвигать для развлекалова дальних родственников на НГ и всё.
Если стулья не магнитные, то похоже что не существует таких полей, которые смогли бы их двигать.
Так что телекинез чисто маняфантазия.

>детонации на луне взрыв гарика снесёт верхний грунтовый слой спутника?
Нет, там же нет воздуха, и ударной волны не будет.
Аноним 05/01/21 Втр 16:43:47 52571970
>>525717
>не существует таких полей
А как же вибрации? Из-за сильно громких звуков предметы колбасит, так что м. б. можно использовать тот-же принцип, но в более деликатной манере.
>ударной волны
Гарик передаст энергию грунту, а грунт это уже среда. +Металлический слой луны под грунтом будет бешено вибрировать.
Вообще я подумал типа: "можно ли снести с луны верхний слой, чтобы люди прям с Земли увидели, что Луна это космическая база".
Аноним 05/01/21 Втр 16:52:04 52572071
>>525719
Если ты уже всё намяняфантазировал в своих маняфантазиях, то зачем задавал вопросы? Чтобы показать какой ты умный?
Нет, это только в твоих маняфантазиях.
Аноним 05/01/21 Втр 16:53:01 52572172
>>525719
Ты как-то помешан на вибрациях.
Любишь долбить себя вибрирующим вибратором?
Аноним 05/01/21 Втр 16:53:20 52572273
Аноним 05/01/21 Втр 17:04:45 52572474
>>525722
Ты задай корректно вопрос. Ты мог с тем же успехом написать "пук кек?". Ветвление мира не быстрее скорости света... что? Ветвление мира обладает координатами? Или у "ветвления" есть скорость? Ты нормально задай вопрос.
Аноним 05/01/21 Втр 17:06:16 52572575
>>525724
> В многомировой интерпретации запутанность никогда не разрушается. Соответственно, при измерении реализуются все возможные варианты (просто в разных мирах). Поэтому измерение на одном конце («щелкаем выключателем») приводит к ветвлению мира на два, в которых реализуются соответствующие варианты событий. Это ветвление происходит со скоростью света, не быстрее.
И почему не быстрее скорости света?
Аноним 05/01/21 Втр 17:26:46 52572876
>>525725
Что значит ветвление не со скоростью света?
Ветвление это не движение, это событие, скорось света это движение.
Что значит ветвление медленее склрости света?

Тебе же говорят, если ты какие-то маняфантазии приносишь, то хотя бы поясняй что там наманяфнтазировано.
Или за тебя/за них надо доманяфаньазировать детали?
Аноним 05/01/21 Втр 17:28:57 52572977
>>525693
Спасибо, эта информация принесла некоторую ясность.
Аноним 05/01/21 Втр 17:43:54 52573078
>>525725
Хуй его знает зачем прилепили скорость света. Вся суть многомировой интерпретации, что мы выкидываем расширяем квантовые состояния на всю вселенную и выкидываем из нее нахуй наблюдателя. "Ветвление" это линейное перепутывание всех квантовых состояний вселенной, для него нельзя задать скорость и тем более сравнить ее со скоростью света, которая является уже отношением отношений уже перепутанных состояний. Короче сравнение теплого с мягким.
Аноним 06/01/21 Срд 06:07:40 52577179
>>525425 (OP)
Ундулиподия это общее название для ресничек и жгутиков или это органоид, который в одних ситуациях ведет себя как ресничка, а в других как жгутик?
Аноним 06/01/21 Срд 08:11:47 52577280
>>525725
Я кажется понял что он пытался спросить. Он спрашивал, почему при многомировой интерпретации сохраняется локальность.

Она там сохраняется, потому что уравнения квантовой механики Лоренц-ковариантны.
Аноним 06/01/21 Срд 09:22:32 52577781
>>525699
Сами сторонники многомировой интерпретации этого не знают. Среди них есть мнение, что ветвление происходит со скоростью света, а есть противоположное мнение. Об этом говорил Шон Кэрролл. Лично я считаю многомировую интерпретацию галимой хуетой, вводящей дополнительные сущности. Вероятностная интерпретация обходится без всяких ветвлений миров и прочей чепухи. В квантовой физике она является абсолютно достаточной, она хорошо объясняет наблюдательные факты при минимуме исходных предпосылок. Я думаю, многомировая интерпретация - это такая отчаянная попытка людей как-то избежать того факта, что миром правит случайность, что мы все играем в одну большую рулетку (редукция волновой функции при измерении в вероятностной интерпретации - это чисто случайное событие). Попытка найти локальное объяснение квантовой физики, убежать от ее нелокальной и случайной природы. Но эта попытка бессмысленна. Реальность нужно принять такой, какая она есть, а не пытаться искать удобные для нашего сознания, но ложные объяснения этой реальности.
Аноним 06/01/21 Срд 09:52:19 52577982
>>525777
>Лично я считаю многомировую интерпретацию галимой хуетой, вводящей дополнительные сущности.
Многомировая интерпретация как раз не вводит никаких сущностей. Она получается, если предположить: "пуcть КМ просто работает на любых масштабах".

>Вероятностная интерпретация обходится без всяких ветвлений миров и прочей чепухи.
Приходится вводить магическое наблюдение. Там причем довольно печально получается, Ньютона механика, является частным случаем КМ, поэтому нам ничего не мешает вообще забить на наблюдение и пользоваться волновой функцией системы. При этом феномен наблюдателя автоматически объясняется.

И потом о каком "ветвлении миров" речь, если подобную риторику используют только в научпоп высерах. У тебя одна волновая функция в любой ситуации, и до измерения и после.

Те, кто хоть раз пользовался многомировой интерпретацией, для объяснения тех или иных эффектов, поражаются лаконичностью таких объяснений.
Аноним 06/01/21 Срд 11:23:00 52579183
Полное или практически полное оледенение Земли, как мотни миллионов лет назад, сейчас больше невозможно?
Аноним 06/01/21 Срд 11:28:16 52579784
>>525791
>мотни миллионов лет назад
Сотни миллионов лет назад, конечно же.
Аноним 06/01/21 Срд 11:31:35 52579985
Есть ли какое-то годное чтиво на тему того, как у людей развивалась способность распознавать образы, строить связи и вот это вот всё, и то как это помогает или мешает им в наше время?
Аноним 06/01/21 Срд 12:21:49 52580586
>>525799
Какая-то херня невнятная, конкретизируй
Аноним 06/01/21 Срд 12:49:01 52580987
>>525799
>как у людей развивалась способность распознавать образы
Я думаю, начать надо с амёб и их образа "свет - нет света".
Аноним 06/01/21 Срд 13:24:58 52582088
>>525791
Если альбедо поверхности повысить на процентов 15-20, то получим Землю-снежок, который сможет оттаять хуй знает когда.
Аноним 06/01/21 Срд 13:28:13 52582189
>>525820
Уже было такое.
Замерзание постепенное и приводит же к увеличению альбедо, всё ж белеет, лол.
Вроде суть саморегулиции при таком что замёрзшая вода, льдом атмосфера от воды отделена, углекислый газ не поглощается и не оседает на дно, но вулканическая активность продолжается, со2 постепенно в атмосфере и всё оттаивает.
Аноним 06/01/21 Срд 13:44:01 52582490
>>525821
У льда как и водяного пара альбедо так себе. Лед непрозрачен для ИК и активно его поглощает. Для повышения альбедо надо чтоб экваториальные области были покрыты сушей, ну еще для верности покрыть ее фольгой.
Аноним 06/01/21 Срд 14:40:28 52582691
Какой Профит можно получить изучая когнитивистику в вузе?
Аноним 06/01/21 Срд 14:41:10 52582792
>>525779
>Приходится вводить магическое наблюдение. Там причем довольно печально получается, Ньютона механика, является частным случаем КМ, поэтому нам ничего не мешает вообще забить на наблюдение и пользоваться волновой функцией системы. При этом феномен наблюдателя автоматически объясняется
Никакого "магического наблюдения" в вероятностной интерпретации нет. Редукция волновой функции происходит всякий раз, когда производится измерение, то есть когда квантовый объект взаимодействует с каким-либо другим объектом (микроскопическим или макроскопическим). При этом взаимодействии разрушается суперпозиция квантового объекта. Измерение же может проводить кто и что угодно, тут вовсе не нужен разумный наблюдатель. Интерференционная картина в двухщелевом эксперименте прекрасно будет получаться и без всякого разумного наблюдателя. Если мы его выкинем из Вселенной - результат двухщелевого эксперимента не изменится, интерференция никуда не исчезнет.

Аноним 06/01/21 Срд 14:47:58 52582993
>>525827
>Редукция волновой функции происходит всякий раз, когда производится измерение
Удачи тебе с экспериментом отложенного выбора, квантового ластика, или неточного измерения. Придется с бубном танцевать, чтобы объяснять, почему при измерении ничего не случилось, если пытаться пользоваться вероятностной интерпретацией. Так еще и локальность на помойку нужно нести. Многомировая же, за три строчки объясняет все эти эксперименты, достаточно внести в уравнение волновую функцию измерительного прибора.
Аноним 06/01/21 Срд 16:21:35 52583694
>>525805
Ну вот есть какая-то болезнь, где человек находит закономерности во всяком мусоре, есть расизм и прочие стереотипы, где человек строит себе картину мира по тому, что видел на работе. Были религии, где люди дрочили на какие-то крысиные норы, думаая, что именно это приносит им урожай.
Ну и вот мне интересно, как это все может быть связанно с примитивным поведением "увидел глаза в кустах - беги"
Аноним 06/01/21 Срд 18:03:41 52584195
Модератор — империалистическая свинья.
Аноним 06/01/21 Срд 18:08:47 52584396
>>525841
Это потому что он пидор и влюбился в уёбка малиновского. А возможно сам малиновский и насасал на модерилку.
Аноним 06/01/21 Срд 19:11:52 52585397
image.png 184Кб, 500x300
500x300
>>525843
Малиновский на доске не появлялся уже с полгода, когда его в последний раз почистили, и даже в своё отсутствие он ухитрился разбить тебе сердце и приковать всё твоё внимание. Я бы задумался, анончик. Я бы задумался.
Аноним 06/01/21 Срд 20:00:41 52585498
>>525853
Как определил? Ты у него в монитор компа пялился?
Аноним 07/01/21 Чтв 01:30:06 52586199
Является ли гравимагнетизм подтверждением связи гравитационного и электромагнитного фундаментального взаимодействия, или это просто наложения одной действующей силы и другой?
И проясните, пожалуйста, электрослабую теорию: и фотон, и Z-бозон, и W-бозоны образованы одними и теми же частицами, но в фотоне и Z-бозоне обе вариации комбинаций тех частиц находятся в суперпозиции, из-за чего они электронейтральны, а вот в W-бозонах такого нет, поэтому бывают и положительно, и отрицательно заряжены? Или последнее через "хиггсовый механизм" происходит? Если да, то что это за "хиггсовый механизм"? В педивикии не нашёл ответа, не отказался бы и от ссылки на литературу.
Почему в последнее время уходят всё дальше от вариаций теории Струн и суперсимметрии? На что тогда засматриваются нынешние исследователи?
Аноним 07/01/21 Чтв 01:33:59 525862100
>>525861
Насколько знаю от теории струн уходят из-за отсутствия возможности проведения экспериментов по доказательству теории (могу быть не прав)
Аноним 07/01/21 Чтв 01:35:43 525863101
Аноним 07/01/21 Чтв 06:39:47 525869102
>>525863
На моменте "наблюдатель на детекторе D0 видит сумму четырех картин распределений" стоит задуматься, почему там вообще две из четырех картинок не имеют интерференцию, если измерение происходит позже, чем фотоны вообще прилетают в D0. Ну и если серьезно, автор не пользуется тут вероятностной интерпретацией он четко проговаривает фразу "как мы помним из какого-то там видео, интерференция не проявляется, если в природе не существует информации о том, через какую щель пролетел фотон", он пользуется информационной интерпретацией квантовой механики. Интересно задачу Элитцура—Вайдмана об испытании бомб он также разбирал. Её вообще тяжело объяснить другими интерпретациями кроме многомировой и информационной.


И в вероятностной есть одна проблема еще в формулировке измерения, "при взаимодействии классического прибора с квантовым объектом реализуется одна из...". При каких размерах прибора он является классическим? Где эта граница? Какая теория расскажет, где проводить границу?
Аноним 07/01/21 Чтв 07:45:42 525871103
>>525861
Гравимагнетизм приближенное решение уравнений ОТО методами из теорию поля(электродинамики). Короче мы считаем гравитацию полем и отбрасываемым из нее искривлением. Это не работает в сильных полях и на космологических масштабах. Связь тут в похожести.

Суть электрослабой теории в том, что у нас есть поля с калибровочной симметрией SU(2) x U. Свойство этой группы симметрии и задают нам свойства квантов поля.
Хиггсовый механизм это введение скалярного поля, которое мягко нарушает симметрию. Нарушение этой симметрии ведет к появлению массы у Z W бозонов. Короче электрослабое поле взаимодействует с неким скалярным полем, из-за они мутирует в то, что мы наблюдаем.
Если хочешь почитать, то накати Вайнберга или кого-нибудь посовременнее. Но это книжки довольно хардкорны.
Струны неоправдали себя, а суперсимметрию не нашли.
Аноним 07/01/21 Чтв 10:48:48 525874104
>>525869
Ну, вообще в квантах же все масштабы считаются квантовыми при S≈hbar
Аноним 07/01/21 Чтв 11:17:16 525876105
>>525874
>≈
В этом и проблема. Теория измерений (выросла в 80-ые вроде) вообще засунула всю эту ебалу под ковер, и просто пользуется матрицей плотности, похуй что там и как, матрица плотности и считай. Классическая или квантовая вероятность, всё в одну кучу смешали и не парятся.

И что-то мне подсказывает, что уже во многих экспериментах чистые состояния наблюдали и при S поболее hbar. Вон тот же LIGO (которым измеряли гравитационные волны), там же в центре стоит тяжеленное полупрозрачное зеркало, так они на нем проверили, можно ли такой объект запихать в состояние суперпозиции.
Аноним 08/01/21 Птн 13:28:55 525934106
Аноним 08/01/21 Птн 15:46:07 525935107
>>525934
Сможешь пососать совковым преподам и прочим дегенератам если ты хуесос и для тебя это "профит".
Аноним 08/01/21 Птн 17:00:12 525936108
>>525935
Узнаю эту злость... Игорь, ты? Помнишь меня?
Аноним 08/01/21 Птн 21:37:52 525948109
Саентач, вот недавно у китайцев из-за ливней переполнилось водохранилище ГЭС Три ущелья, пришлось стравливать воду и эвакуировать население, и все в интернетах винили в этом тупых китайцев которые строят лишние небезопасные плотины.
Но разве без этой плотины их не затопило бы? Водосток ведь в это время всё равно был бы такой же, только не было бы ещё регулирующего фактора плотины
Аноним 08/01/21 Птн 22:21:43 525949110
>>525948
Их всю историю это говно затапливало
Аноним 08/01/21 Птн 23:31:54 525955111
>>525936
Ты почему порвалась, манька-хуесоска?
Аноним 09/01/21 Суб 00:41:22 525962112
>>525716
>эта концепция представляется исключительно магической?
Да. Охуенно. Еще бы она не представлялась магической. В Саентаче то.
> Я вот думаю, что если делать тело человека плотнее и сильнее, то в результате оно тупо окаменеет.
Ага. А если их складировать штабелями и уплотнять, можно нейтронную звезду скрафтить.
>А телекинез это вообще чистая фантастика
Не совсем. Изначально это производное от парапсихологии. Парапсихология провалилась окончательно давным-давно.
>телекинез и может быть, генерируясь посредством специального органа, то он явно не будет суперменского уровня
Почитай лучше про электрических угрей. Вот они могут. Хоть и не в телекинез.


Аноним 09/01/21 Суб 00:42:01 525963113
>>525949
Так а какого хуя в ёбаных СМИ плотина подаётся как зрада?
Аноним 09/01/21 Суб 00:43:48 525964114
>>525963
По той же причине. По которой Роскосмос в публиковал в 2018 году в СМИ "уникальные снимки с Венеры", слитые в 2003. Новостники ищут хайпа и контента. Им ваще похуй. Даже если новость 20 летней давности. Если это выстрелит. Это нужно срочно в номер.
Соотношение полов Аноним 09/01/21 Суб 11:20:35 525977115
geneshaft1877x2[...].jpg 1300Кб, 1877x2674
1877x2674
Наверняка в треде хоть кто-то смотрел аниме Geneshaft. В предыстории мира там есть такой эпизод: методами генной инженерии, ЭКО и т.д и т.п было сделано так, что теперь на 9 женщин приходится всего 1 мужчина. Вопрос: к чему приведёт настолько радикальное изменение соотношения полов?
Аноним 09/01/21 Суб 11:43:32 525978116
>>525977
Скрепы треснут ("традиционная" семья и подобный бред) а так-то ничего особенного. Тем более если это будет "генной инженерии, ЭКО" а не геноцид или что-то подобное (катастрофичекое). У общества будет дохуя времени подстроится. Люди может даже и не очень заметят перемены, только деды будут бурчать что раньше по другому было.
Аноним 09/01/21 Суб 11:43:47 525979117
>>525977
Гаремы и всё такое. Рождаемость станет пониже, но не критично ниже. Фертильность в популяции не особо зависит от количества мужчин.
Аноним 09/01/21 Суб 11:46:18 525980118
>>525977
Да ни к чему хорошему. Даже если забыть, что разнообразия стало меньше (что на долгосрочную перспективу полный айайай). Будет эдакий лисий прайд (альфа-пара). Так как мы в ИРЛ, то наверняка будет и побочное потомство. Но обязательно накопится критическая масса женщин, которая не поучаствует в размножении. Что будет дальше, можешь представить. Это мы даже не затрагиваем вопрос того. Что для такого общества победившего феминизма придется в буквальном смысле евгенику возвращать и общую биологию бустить лет так на 250 вперед. Иначе траблов не оберётесь. И речь даже не про способы уничтожения мужского пола на самом первом этапе. Речь о генетике будущих поколений.
Аноним 09/01/21 Суб 11:51:33 525981119
>>525978
>Тем более если это будет "генной инженерии, ЭКО" а не геноцид или что-то подобное (катастрофичекое).
Я правильно понимаю коллега. Что если мы изобретем метод, согласно которому XX будут рождаться только 1 раз на 9 случаев. Вы не назовёте это геноцидом и это приемлемо?
Аноним 09/01/21 Суб 11:54:03 525982120
>>525981
> согласно которому XX будут рождаться только 1 раз на 9 случаев. Вы не назовёте это геноцидом и это приемлемо?
Вернее переходить в XY будут 8 случаев из 9. А 1 вариант будет оставаться XX. Ну женщинок крч уничтожим, будет только одна рождаться. Практические изыскания уже есть. Китай тому пример. Если девочка, за борт и аборт. Мальчиков оставляем. Даже ничего толком изобретать не придется. Это ведь не геноцид, верно?
Аноним 09/01/21 Суб 12:54:41 525989121
>>525981
>>525982
А зачем уменьшать количество женщин? Их должно быть больше мужчин хотя бы на треть, причём для блага самих мужчин.
Аноним 09/01/21 Суб 12:57:04 525990122
>>525989
>А зачем уменьшать количество мужчин? Их должно быть больше женщин хотя бы на треть, причём для блага самих женщин.

Выбора же больше лол.
Аноним 09/01/21 Суб 13:15:15 525994123
>>525990
Вот именно, у мужчин появится выбор женщин, исчезнут инцелы.
Аноним 09/01/21 Суб 13:16:53 525995124
>>525994
Чего вот именно. Ты ваще читаешь что написано?) Я повторю.

>А зачем уменьшать количество мужчин? Их должно быть больше женщин хотя бы на треть, причём для блага самих женщин
Аноним 09/01/21 Суб 13:43:24 525999125
15997432273200.jpg 147Кб, 604x531
604x531
Аноним 09/01/21 Суб 14:32:57 526001126
>>525999
Ну вот и закончилась маня-аргументация. Перешли на картиночки. Эх.
Аноним 09/01/21 Суб 14:53:18 526002127
>>526001
Какой уровень дискуссии, такие и аргументы а не наоборот, кек
Аноним 09/01/21 Суб 15:16:03 526003128
>>526002
Уровень дискуссии всегда можно повысить или снизить. Аргументацией) Так что не передергивай. Лучше иди штурвал передергивай или чо у тебя там.
Аноним 09/01/21 Суб 16:53:11 526008129
Двачик, помоги найти график длины свободного пробега фотона в воздухе при н.у. в зависимости от длины волны, для фотонов с длиной волны около 555 нм.
Аноним 09/01/21 Суб 17:51:43 526013130
image 212Кб, 1354x712
1354x712
image 322Кб, 1354x712
1354x712
Загадка жака... портящихся фруктов.

Заметил что стали быстро покрываться плесенью/загнивать фрукты на подоконнике. Буквально 2-3 дня и уже несъедобные. Окно на балкон не плотное, сверху слегка задувает, есть приток холодного воздуха со стеклянного балкона. Под подоконником батарея очень горячая, невозможно держать руку. Может ли быть что из-за жара батареи фрукты портятся с увеличенной скоростью?
Раньше такого не замечал, портиться стали в 2019 году, особенно заметно с наступлением осени.
На вскидку правильно я нарисовал потоки воздуха?

Фрукты убрал в другое место.
Аноним 09/01/21 Суб 19:42:29 526014131
>>526013
>Заметил что стали быстро покрываться плесенью/загнивать фрукты на подоконнике. Буквально 2-3 дня и уже несъедобные. Окно на балкон не плотное, сверху слегка задувает, есть приток холодного воздуха со стеклянного балкона. Под подоконником батарея очень горячая, невозможно держать руку. Может ли быть что из-за жара батареи фрукты портятся с увеличенной скоростью?

Да, может. Я те больше скажу. В общагах, когда вывешиваешь пакет со жратвой за окно. Портится всё заметно быстрее. Просто из-за того, что даже при -15 за окном. Определенное влияние оказывает тепло, исходящее от окон. Чего уж там про комнату говорить с батареей.
Аноним 09/01/21 Суб 21:45:45 526035132
>>526008
Для фотона длину свободного пробега считать бессмысленно, потому что у фотона нулевое собственное время. Вместо этого используют параметры рассеяния. В твоих условиях будет рэлеевское рассеяние, там будет угол и поляризация. А они уже зависят от параметров источника света и среды.
Аноним 09/01/21 Суб 21:55:57 526038133
>>526035
>Для фотона длину свободного пробега считать бессмысленно, потому что у фотона нулевое собственное время
Схерали, мне же не для него а для себя. Похер как он там в своей системе отсчёта мнгновенно телепортируется в искажонном простанстве-времени.

По факту в реальности для меня как наблюдателя он же какое-то расстояние пролетает.
Аноним 09/01/21 Суб 22:18:59 526044134
image.png 453Кб, 800x558
800x558
Вот есть эффект Даннинга-Крюгера, где грят, мало усилий дадут лишь ощущение превосходства, а результата нет и при этом есть принцип Паретто, где 20% усилий даст 80% результата. Вопрос: кому верить или в какую сторону исправить мое понимание, в чем я не прав?

А то чет сейчас мне думается, что придумать что-то, конечно, сложно, но среди знаний человечества нет таких направлений особых, чтобы за месяц изучения не освоить 80% или примерно такие сроки. Вот уже собственно эти 20% то да, надо оттачивать долго и нудно.
Аноним 09/01/21 Суб 22:21:08 526046135
>>526044
Или эффект Даннинга-Крюгера это просто о том, что надо быть критичным к себе и не зазнаваться?
Аноним 09/01/21 Суб 22:36:28 526048136
>>526044
>таких направлений особых, чтобы за месяц изучения не освоить 80% или примерно такие сроки
1)тут как посмотреть, если просто считать 80%-ми "знать в общих чертах", то да, но если считать в выполнении реальной задачи-то нет, везде очень много тонкостей, ты как бы знаешь в целом как что-то работает, но в каждой милипиздрической составляющей по 100 тонкостей, в итоге нихуя никакую реальную задачу ты выполнить нн смодешь, хотя вроде как в общих чертах всё знаешь.
Простой тебе пример, электроника, в ней штук 100 разных возможных компонентов.
Но на каждый 1 этот жалкий компонент есть целые разработческие и научно-исследовательские отделы, которые ебутся только исключительно над одним этим компонентом. Они там не просто так сидят.

2)Смотря как повезёт с источником информации, ты можешь прочитать за 1 минуту то что надо и доходчиво будет объяснено, а можешь 1 месяц потратить чтобы просто найти об этом ноомальную инфу. Тут как повезёт.
Аноним 09/01/21 Суб 22:40:08 526049137
>>526044
Плюс пока ты не найдёшь эту нормальную инфу, ты зачастую просто не будешь знать что ты дохуя времени тратишь впустую на некачественно составленную информацию.

А даже если и поймёшь что говно и надо бы что-то нормальное найти. А как? А хуй знает, ты ж не знаешь то что не знаешь, и как оно должно быть по-нормальному про это объяснено и где, нихуя не поможет и будешь продолжать рыться месяцами в поисках каких-то тонкостей.

Ну или нанимать личного эксперта, который будет лично и сразу отвечать на любые твои вопросы.
Аноним 09/01/21 Суб 22:42:20 526051138
>>526038
Длина свободного пробега актуальна только в классическом случае частиц без внутренних параметров.
Фотон нельзя в приближении упростить до точечной частицы без внутренних параметров. Если их выбросишь, то для него потеряешь параметры движения. Поэтому столкновение фотона у тебя будут всегда волновыми, а это уже называют рассеиванием.
Аноним 09/01/21 Суб 22:45:54 526052139
>>526051
>Поэтому столкновение фотона у тебя будут всегда волновыми, а это уже называют рассеиванием.
Но ведь можно длину свободного пробега из этого выявить?
Как по определению-до первого столкновения.
Похер какое потом будет после него, волновое, не волновое...
Аноним 09/01/21 Суб 23:01:34 526055140
>>526052
А как ты собрался искать длину свободного пробега? Расстояние между источником и приемом? А как ты будет определять что именно тот фотон, а не другой? Эти все проблемы решались, если бы у фотона было собственное время. Да зачем тебе длина свободного пробега, если у тебя амплитуда рассеяния, которая дает внятные данные.
Аноним 09/01/21 Суб 23:20:40 526057141
>>526055
Не понял как на это собственное время влияет, мы же сторонний наблюдатель.

Среднестатистически.
Например выпустили n фотонов, столько-то долетело до детектора.
Ну или по из этого рассеяния вычислить.

Меня что интересует, вроде длина среднего пробега фотона даже низких энергий-видимого спектра, 555нм допустим, в воздухе очень мала. Я раньше находил данные-несколько сантиметров, но сейчас чё-то не могу найти.
Так вот, после столкновения с атомом/молекулой из воздуха, каждое такое столкновение это ведь поглощение электроном и переизлучение в случайном направлении?
Если это так, то не понимаю как рассеняие в воздухе такое слабое, и как скорость света в воздухе так мало замедляется.
Ароде как если это всё так-средняя длина свободного пробега несколько сантиметров, а после этого-поглощение и переизлучение в произвольном направлении, то оно буквально за несколько метров должно было в разные стороны разлетаться.
Даже если бы там как-то "тренд стремился сохраняться" по движению в том же направлении, среднестатистическии опять же-таки,
то с учётом среднего свободного пробега несколько см, и переизлучении в случайном направлении-ну не могли бы лазеры бить на десятки метров и даже сотни и километры по прямой, а просто конусом широким становились бы за несколько десятков метров.

Ну и скорость света должна была бы замедляться сильнее.
Аноним 09/01/21 Суб 23:23:33 526058142
Намного сильнее, в разы, а не 1.03 вроде коэффициент там в воздухе или сколько в среднем для видимого спектра..
Аноним 10/01/21 Вск 00:57:36 526065143
>>526057
>n фотонов летело - сколько-то прилетело.
по-честному у тебя нет такой величины, как длина свободного пробега для волны. Только "расстояние ослабления амплитуды на децибел", а это уже гуглится. Ну, или дай больше конкретики, что именно тебе нужно)
Напоследок - у тебя фотоны не поглощаются с каким-то шансом, а пучек перераспределяет амплитуды для своего состояния. Если будешь считать только кол-ва фотонов, то это незаметно, но есть способы это отличить. Правда это уже в залупу лезу.
Аноним 10/01/21 Вск 00:59:28 526066144
>>526058
этот эффект не из-за замедления времени и прочей СТОшной магии, иначе у тебя и фазовая скорость уменьшалась, а тут она наоборот увеличивается
Аноним 10/01/21 Вск 02:22:42 526069145
>>526065
>по-честному у тебя нет такой величины, как длина свободного пробега для волны.
Ну это бред же, фотон это не волна во всех направлениях, по крайней мере видимого спектра, это довольно сильно локализованная в пространстве штука.
Он не как волна на воде, а как бугорок на воде.
Несмотря на все маняфантазии типа "пока фотон не попал в цель он может быть где угодно!! он находится везде и нигде одновременно!!"

Он вполне локализован, и летит по вполне себе траетории, до столкновения прямой(если не искажено пространство)
И это доказывается измерениями, по крайней мере среднестатистическими.
Аноним 10/01/21 Вск 02:24:06 526070146
>>526065
>Напоследок - у тебя фотоны не поглощаются с каким-то шансом
Это не понял.
Ты про каждое столкновение, типа не всегда шанс.
Или про полёт фотона от начала до конца, типа не все сталкиваются?
Аноним 10/01/21 Вск 02:25:27 526071147
>>526065
>а пучек перераспределяет амплитуды для своего состояния
Ну не знаю, ты про то что типа рассматривается пучок фотонов как одна цельная волна?

Но нет же, это в реальности много отдельных фотонов.
Аноним 10/01/21 Вск 05:49:13 526079148
>>526071
Не-а. Пучок фотонов это именно одна волна, просто она описывается в координатах амплитуда-импульс, причём амплитуда комплексная. >>526070
Я про то, что состояние очень слабо декогерирует. Тип если пустить Параллельно пучки света, а потом посмотреть на интерференцию, то она не будет теряться при увеличении пути, при этом будет ослабевать по интенсивности. При этом если бы они вероятностно поглощались, то это картина бы ломалась.
>>526069
Локализованый свет есть только в волноводах и резонаторах. Пучок лазера "чувствует" коэф преломления даже за пределами ширины пучка.
Аноним 10/01/21 Вск 12:34:30 526084149
>>526079
>Локализованый свет есть только в волноводах и резонаторах. Пучок лазера "чувствует" коэф преломления даже за пределами ширины пучка.
Ага, типа летит пучок лазера в вакууме, пролетает возле куска стекла, и преломился
Аноним 10/01/21 Вск 12:38:11 526085150
>>526079
>Пучок фотонов это именно одна волна, просто она описывается в координатах амплитуда-импульс, причём амплитуда комплексная
Ну это её описывать можно как цельную, а в реальности это куча отдельных фотонов, светодиод их же по одному выпускает(не в единицу времени типа, а из разных своих частей)
И я уверен это можно как-то доказать, на основе самого пучка.
Дифракции это уже отсев разных волн по длине, и после этого отсева ещё можно как-то отдельные фотоны отсеять.
Аноним 10/01/21 Вск 12:45:50 526087151
>>526079
>Локализованый свет есть только в волноводах и резонаторах.
Локализованность фотонов доказывается измерениями, его размер-порядка длины волны.
Как раз тот самый "бугорок" перемещающийся, перемещающееся 3-х мерное овальное возмущение поля.
Аноним 10/01/21 Вск 13:52:28 526093152
>>526044
Принцип Паретто работает всегда. Грубо говоря, вот работал ты такой два дня, и понимаешь, что 80% работы сделал за 4 часа. Думаешь, ну ладно, в следующий раз буду работать только 4 часа и сделаю 80% работы. Работаешь 4 часа и понимаешь, что сделал в пять раз меньше чем за 2 дня, но за эти 4 часа, большую часть работы ты сделал за 48 минут.

Короче принцип просто показывает что-то, но использовать его не выйдет. Замени в моем рассуждении "работу" на "освоение знаний" и поймешь, что твои рассуждения про "быстрое освоение 80%" полная хуйня.
Аноним 10/01/21 Вск 14:10:10 526098153
>>526093
>Думаешь, ну ладно, в следующий раз буду работать только 4 часа и сделаю 80% работы. Работаешь 4 часа и понимаешь, что сделал в пять раз меньше чем за 2 дня
Значит в прошлый раз это было не 80% работы, очевидно.
Аноним 10/01/21 Вск 14:30:44 526100154
>>526098
Нет, просто ты не можешь повысить концентрацию полезного времени. Вот и всё.
Аноним 10/01/21 Вск 16:10:56 526106155
>>526100
Тупость.
Количество работы=трудозатраты.
Больше трудозатрат, значит больше работы было.
Аноним 10/01/21 Вск 16:13:39 526107156
>>526106
Да, но всё-равно 80% времени будут там напрямую делать только 20% пользы.
Аноним 10/01/21 Вск 16:16:03 526109157
>>526107
>20% пользы
Чисто шизоидная фантазия и вера в цифры и в авторитета. Ведь ты не сможешь внятно нормально объяснить почему измерил именно 20%.
Аноним 10/01/21 Вск 16:17:54 526110158
Так же как и слепая вера в цифры и в авторитета насчёт "20% людей делают 80% работы"

А если оплата сдельная? А если нормы выработки есть? Те кто за 80% времени делал 20% работы просто уволен нахуй, и остались только те кто делает +-одинаково.
Аноним 10/01/21 Вск 16:57:33 526113159
>>525425 (OP)
Где-то видел такую информацию, что функциональное деление правого и левого полушарий мозга, вместе с Роджером Сперри, обоссали. Интересует насколько обоссали и какая альтернатива. Но чтобы изложено максимально просто, так как я быдло и поэтому туп и ленив, и просто тешу свое любопытство.
Аноним 10/01/21 Вск 17:59:52 526120160
>>526113
>функциональное деление правого и левого полушарий мозга, вместе с Роджером Сперри, обоссали.
Анон, дело в том, что грубо говоря, еще лет 50 назад. Появились практические доказательства несостоятельности "фундаментального деления" полушарий, зон головного мозга и прочих традиционных и вполне логичных теорий. Карлос Родригез, Джоди Миллер и это только те случаи, что на поверхности. В Ланцете куча статей на эту тему по пациентам, чьи имена не раскрываются (пример такой статьи https://www.thelancet.com/journals/lancet/article/PIIS0140-6736(07)61127-1/fulltext).

На данный момент, уже экспериментально подтверждено, что полушария могут брать на себя функции друг друга, или пахать сразу за 2 полушария и т.п. Да, в отдельных случаях траблы все-таки заметны (ну кроме тех, когда у чувака визуально пол-башки не хватает). Но в целом "статичной привязки" на полушария нет. Она как минимум динамическая.
Аноним 10/01/21 Вск 18:08:01 526122161
>>526120
Ну, мозг - очень пластичный орган, это уже хорошо известно.
Аноним 10/01/21 Вск 18:27:27 526125162
>>526120
Корректировка https://www.thelancet.com/journals/lancet/article/PIIS0140-6736(07)61127-1/fulltext (скобка с точкой присобачились,ссылка битая была).
Вот раскрытие этой ситуации весьма доступно и на русском
https://habr.com/ru/post/395965/

>>526122
Ну да. По теме теорий (новых) с учетом исключающей практики, я не силён. Потому что отталкиваться можно от концепции (ну если пол-башки не снесли, то статично, а если снесли. то пластичность). Надо других анонов ждать, мб чо подкинут на эту тему, чего там нового натеоретизировали
Аноним 10/01/21 Вск 21:44:19 526135163
>>526087
Фотон это пакет волн. "Бугорок" это движение для среднего. Отклонение от среднего для одиночного фотона при измерении могут быть очень даже макроскопическими.
Аноним 10/01/21 Вск 22:26:33 526137164
>>526135
>Фотон это пакет волн.
Схерали? Один электрон при переходе с одного уровня на другой разве выпускает не строго один фотон?
Аноним 10/01/21 Вск 22:29:52 526139165
>>526135
>Фотон это пакет волн
И почему ты считаешь его фотоном, а не несколькими фотонами?
Аноним 10/01/21 Вск 22:44:02 526140166
>>526109
Чел я не просто так написал "напрямую". В этом проблема правила Паретто
Аноним 10/01/21 Вск 22:45:58 526141167
>>526140
Ну это же просто слово с маняфантазийным определением, которое которое остальным забыли рассказать.
Аноним 10/01/21 Вск 22:49:51 526143168
2018-12-2013-48[...].png 186Кб, 585x612
585x612
>>526071
>>526079
>>526087
>>526139
Вы что там курите?
>И почему ты считаешь его фотоном, а не несколькими фотонами?
Потому что суммарная вероятность там ровно на 1 фотон. Просто там суперпозиция волн с разной длиной волны, грубо говоря чем больше локализован фотон в пространстве, тем большая неопределенность по его длине волны (пакет раплывается). Если тебе нужен фотон именно с конкретной энергией-импульсом, то это одна конкретная бесконечная в обе стороны хуйня
Аноним 10/01/21 Вск 22:53:22 526144169
>>526143
Ну хз, сомнительно что это подтверждается экспериментами, а скрин из книги так вообще хуета, там чисто одно утверждение, без доказательств.
Это вообще хуета, я тоже могу напечатать на бумажке текст "какашка моего котика-новый иисус", это то же самое что и на скрине том утверждение.
Аноним 10/01/21 Вск 22:55:33 526145170
>>526137
Если у тебя атом один во вселенной, находится в чистом возбужденном состоянии, испускает фотон в канал с нулевой температурой, да и наблюдение состояния ты проводишь через половину периода осцилляции Раби, то да. Если взять реальные источники(где атом не один, заселенность не полностью инвертированная, канал не при нулевой температуре и прочее), то вместо одного фотона у тебя будут генерироваться пакеты когерентного излучения. Да если и сможешь отправить один фотон, то при прохождении через среду он будет оставаться чистым состоянием, без всяких вероятностей.
Аноним 10/01/21 Вск 22:59:59 526146171
>>526145
Так это же просто куча разных фотонов, зачем их считать одним фотоном с пакетом разных волн?
Аноним 10/01/21 Вск 23:05:16 526147172
>>526144
>сомнительно что это подтверждается экспериментами
Лазер теперь не лазер, пацаны) Слышали чё?)

Вообще если серьезно, можешь считать за точку отчета вариант, где фотон локализован в точке, но тогда этим не очень удобно пользоваться, неопределенность энергии бесконечна. Тот фотон, который излучается из атомов (как в лазерах, да) он именно что разъебан в пространстве, чем точнее у атома дискретные уровни, тем более разъебан (да, у атома уровни не бесконечно тонкие, они там немного расплывчатые, можно это объяснить тем, что время жизни возбужденных состояний не бесконечное, а еще математически это аналогично влиянию виртуальных фотонов). Короче, ты просто пытаешься спорить с определением, просто за определение фотона взяли именно размазанный в пространстве случай, как самый удобный и как более близкое к тому, что выблевывает атом, когда у него меняется состояние.
Аноним 10/01/21 Вск 23:10:31 526149173
>>526147
Так а чё точку, я же говорю-размер порядка длины волны.


Так а что, атом выпускает за один "такт" сразу кучу фотонов, но их принято считать одним фотоном?
Аноним 10/01/21 Вск 23:24:47 526150174
>>526149
>говорю-размер порядка длины волны.
Нет, намног.... намного больше может быть. Обычный лазер 550 нм, там фотон примерно см 20-25 (если вообще конечно имеет смысл говорить о размерах фотона, это немного глупо).
>Так а что, атом выпускает за один "такт" сразу кучу фотонов, но их принято считать одним фотоном?
Не совсем, он просто типа срет вероятностью вокруг среднего значения длины волны. Грубо говоря, у тебя атом выплюнул фотон, и фотон этот немного разъебан в своей энергии.

Блин, короче там от классических вещей вообще ничего уже не осталось, понять что там происходит можно только разобравшись в формулах. Я не могу тебе аналогиями из классики пояснить, этих аналогий нет.
Аноним 10/01/21 Вск 23:25:34 526151175
Вопрос про эволюцию.
Как эволюционно может появиться страх перед какими-то ядовитыми насекомыми или еще чем-то смертельным и вредным? Как это само по себе может возникнуть в виде мутации, если в течение жизни, когда особь сталкивается со средой, гены типа не должны меняться (если мы не глупые ламаркисты)? Типа либо ты рождаешься и живешь как хочешь и лезешь мордой к ядовитым паукам, и умираешь, либо по какой-то причине не лезешь к паукам и выживаешь - но откуда у потомства возьмется страх пауков?
Аноним 10/01/21 Вск 23:30:46 526152176
>>526151
Короче если тебя укусил паук и ты выжил и твое потомство стало бояться пауков — ламаркизм в какой-то степени верен?
Если вообще не брать вариант с выживанием после укуса паука — ты получается должен передавать страх к паукам тупо от созерцания того, что паук убивает кого-то другого? (если он тебя укусил и ты слишком поздно понял, что пауки плохо, ты ведь не оставишь потомства и ничего им не передашь)
Аноним 10/01/21 Вск 23:47:45 526153177
>>526150
>Блин, короче там от классических вещей вообще ничего уже не осталось, понять что там происходит можно только разобравшись в формулах. Я не могу тебе аналогиями из классики пояснить, этих аналогий нет.
Так зачем аналогиями, ты говори что датчики показывают если знаешь о таких измерениях, мне это не надо аналогии и не аналогии, мне чисто что по факту датчики показывают.

>Грубо говоря, у тебя атом выплюнул фотон, и фотон этот немного разъебан в своей энергии.
И как это по показаниям датчика определили?
Явно же чередой замеров, но это же не значит что каждый фотон был неопределённой энергии, они просто были разных энергий.
Если нет, то как определили что именно один фотон но разъёбан в энергии? Замерами, понятно что придумать можно всяких разных теорий, и они даже не будут противоречить измерениям...

>>526150
>Нет, намног.... намного больше может быть. Обычный лазер 550 нм, там фотон примерно см 20-25 (если вообще конечно имеет смысл говорить о размерах фотона, это немного глупо).
Опять же, очень сомнительно.
Если взять отверстие 0.1мм, и светить через него лазерным лучом 0.09мм с длиной волны 555нм, то он вообще никак ослабляться не будет, а если бы реально был размер 20-25 см, то через отверстие проходило бы около НУЛЯ фотонов, т.е. лазер бы вообще не мог просветить через отверстие.
Если ты исключительно про его длину, то ок, учитывая скорость фотона-его такая длина это похуй, доказать сложно и ни на что почти не влияет.
Аноним 11/01/21 Пнд 00:22:20 526155178
>>526153
Конкретика твоих вопросов уже дошла до уровня, что тебе будет проще самому открыть учебник. Я серьезно. на этом этапе я начну тебе просто его словами пересказывать, когда проще самому уже прочитать. Начни с обычных квантов.
Аноним 11/01/21 Пнд 01:38:13 526157179
Аноним 11/01/21 Пнд 01:40:18 526158180
>>526157
Хм... А какой у тебя уровень подготовки? За гамильтову механику пояснить можешь? Скобки Пуассона? Линал знаешь? Собственные вектора найдешь?
Аноним 11/01/21 Пнд 02:07:46 526159181
>>526158
Ну ты же видел вопросы, мне бы про измерения почитать, а не фантазии физиков и математиков основанных на этих измерениях... тем более в виде формул.
Аноним 11/01/21 Пнд 02:59:07 526163182
>>526159
Тогда это будет немного бессмысленно. Ты наверное считаешь, что что-то поймешь. Спешу тебя разочаровать, твоя голова не сможет родить ни одной достойной интерпретации результатов. Кванты в таких муках рождали, под взрывы пердаков реалистов.
Аноним 11/01/21 Пнд 03:15:25 526164183
>>526163
Вполне себе всё понятно будет по результатам измерений.
Так есть что такое почитать без маняфантазий формульно-математических? Чисто про замеры
Аноним 11/01/21 Пнд 04:10:17 526172184
>>526164
Если хочешь экспериментальных квантов, то гугли квантовую оптику.
Впрочем раз ты задаешь такие вопросы, то ты сломаешься на опытах с поляризаторами, хотя это считай начальный уровень.
Аноним 11/01/21 Пнд 04:25:48 526176185
>>526172
Но мне нахуй не нужны опыты с полиризаторами, мне нужны измерения размеров единичных фотонов и о их движении в средах.
Аноним 11/01/21 Пнд 04:44:00 526179186
.webm 496Кб, 1280x720, 00:00:10
1280x720
Моё лицо от последних 10-ти постов в этом треде
Аноним 11/01/21 Пнд 04:48:30 526181187
>>526179
Ты тупой и не понял что в них было написано?
Аноним 11/01/21 Пнд 05:04:58 526184188
>>526181
Удивляюсь, что кто-то всерьез что-то отвечает человеку, который хочет узнать про размер фотона, но не понял с первого раза, что это бессмысленное понятие.
Аноним 11/01/21 Пнд 05:38:13 526187189
>>526184
Думаю ты просто слишком тупой и даже не вникал почему это якобы бессмысленно, просто прочитал один раз фразочку "это бессмысленно" и принял на веру.
Аноним 11/01/21 Пнд 09:55:23 526202190
>>526144
>Ну хз, сомнительно что это подтверждается экспериментами, а скрин из книги так вообще хуета, там чисто одно утверждение, без доказательств.
>Это вообще хуета, я тоже могу напечатать на бумажке текст "какашка моего котика-новый иисус", это то же самое что и на скрине том утверждение.

Т.е. ты уже склоняешься к той версии, что аноны в Саентаче в качестве пруфов подделывают целые страницы учебника, чтобы потом каких-нибудь дебичей затыкать? Тебе не кажется это странноватым? Сама версия.

мимоанон
Аноним 11/01/21 Пнд 09:55:45 526203191
>>525448
Это лютая смесь углерода и обломков углеродных цепочек от органики, связанная в плотный слой. Фэйри не растворяет так как он может растворять только собственно жиры, а там обгоревший кал практически.
Аноним 11/01/21 Пнд 10:08:38 526205192
>>526187
Да-да, я слишком тупой. Не расстраивайся. Все слишком тупые.
Аноним 11/01/21 Пнд 14:04:48 526235193
>>526202
Там было чисто одно утверждение, без доказательств нормальных.
Учебник может быть говном полным, и шизой полной может быть, ему ни что не мешает таким быть.

И книжки якобы учебные религиозных дегенератов печатают, и прочая хуйня творится.
Аноним 11/01/21 Пнд 14:06:11 526236194
>>526205
Ну не расстраивайся(( может поумнеешь потом и перестанешь в АВТОРИТЕТОВ сразу верить и под них стелиться бездумно.
Аноним 11/01/21 Пнд 15:05:14 526240195
image.png 1088Кб, 800x640
800x640
>>526236
>Ну не расстраивайся(( может поумнеешь потом и перестанешь в АВТОРИТЕТОВ сразу верить и под них стелиться бездумно.
Аноним 11/01/21 Пнд 15:14:09 526241196
>>526236
Так я сам понимаю почему понятие размер фотона бессмысленно. Но для этого нужно сначала понять, как работают операторы в квантовой механики и т.д. Если ты не можешь посмотреть курс из 30-ти лекций чтобы разобраться самому, то проблема наверное в твоей лени, раз считаешь, что можно без знаний понимать сложные вещи.

Или ты хотел чтобы тебе бесплатно пришли и стали объяснять? Ну пиздец. Массовая культура рождает миф, что умные знают физику, а потом ущемленное в самомнении быдло пытается сразу лезть в вещи, в которые им еще рано лазить, потому что убеждены что они все неебаца Энштейны и всё сами смогут понять.
Аноним 11/01/21 Пнд 19:19:50 526250197
free.jpg 209Кб, 1200x799
1200x799
сап
такое дело. видел по науч попу какой-то эксперимент или опыт, где в общем есть ИИ машина, которая должна дублировать произнося вслух только истинные изречения. Типа говоришь что у человека 5 пальцев на руке, она повторяет это. Но физик ее запутал какими-то словосочетаниями напичканными частицей НЕ. в общем не помню. но суть в том что она повторила, хотя это было очевидно ложное изречение. типа он ее сломал и вынудил солгать.
может кто понял про что я говорю. т.к. я и не помню какой вывод он сделал на этом
Аноним 11/01/21 Пнд 20:58:36 526260198
Аноним 11/01/21 Пнд 22:41:53 526267199
>>526241
>Но для этого нужно сначала понять, как работают операторы в квантовой механики и т.д.
Нет не нужно, шиз, нужны результаты измерений, а не маняфантазии всякие.
Аноним 11/01/21 Пнд 23:58:55 526275200
>>526203
И чем химически такую хуйню убрать?
Аноним 12/01/21 Втр 00:01:39 526277201
>>526275
Химическим покрытием стального ворса латунью и вращением щётки из такого прута с высокой скоростью.
Аноним 12/01/21 Втр 03:39:20 526279202
>>526267
Нужны измерения? Эксперимент сам поставишь - берёшь интерферометр маха-цедера и после первого зеркала помещаешь нейтральные фильтры. Посылаешь в схему пучки света амплитудой, чтоб после фильтра было состояние, в котором можно было пользоваться приближением одного фотона, а потом смотришь на то, есть ли интерференция. Если бы поглощение было вероятностным, то после фильтра было бы случайное кол-во фотонов в каждом из каналов и интерференцию бы не было видно. Эксперимент в такой постановке я не видел, но теория говорит, что интерференция останется.
Аноним 12/01/21 Втр 10:39:21 526293203
>>525448
>Почему эту фигню ничем не растворить?
Ничем не заполнить пробелы в твоем образовании.
А коксование легко устраняется сверхкритической водой или ультразвуком.
Аноним 13/01/21 Срд 06:55:29 526347204
Все ли породы собак скрещиваемы между собой или для каких-то есть репродуктивная изоляция? Той-терьера с мастифом например
Аноним 13/01/21 Срд 10:36:55 526356205
>>526347
Генетически они все скрещиваемы. Потому что это один вид. А вот чисто инженерно, ну сам понимаешь. Естественным путём ты тибетского мастифа с мопсом не скрестишь. Да лучше и неестественным не пробовать.
Аноним 13/01/21 Срд 12:20:51 526358206
>>526347
Все разнообразие пород собак является полиморфизмом нежели генетическим разнообразием. Если кратко, то незначительные генетические изменения через эпигенетические механизмы могут создавать заметное различие в фенотипе.
Почти все породы собак дают плодовитое потомство с волками, а волки имеют самую большую генетическую дистанцию.
Аноним 13/01/21 Срд 12:48:10 526360207
У меня вопрос.
Простой и тупой, но что-то я ответа не могу найти, так что может двачеры подскажут.
Вот есть магнит с редкоземельными металлами. У него остаточная намагниченность очень высокая. Ну по сравнению с куском стекла, например.
Общеизвестно, что магнитное поле порождается движущимися зарядами. В постоянном магните, якобы - циркулирующими токами.
Чтобы что-то материальное, в том числе электрон, начать двигать, надо сообщить ему энергию.
Так вот мой вопрос в том, откуда в постоянном магните берется энергия, которая позволяет ему терять всего 2-3 процента коэрцетивной силы за 10 лет при нормальном режиме работы? Как современная наука это объяснят?
Аноним 13/01/21 Срд 13:25:43 526361208
>>525425 (OP)
Почему никто нормально не вкладывается в евгенику? Почему никто не пытается ввести сверхумных людей где-нибудь на отдаленной базе, чтобы потом из воспитать для продвижения прогресса? Ведь рили, интеллектуальный потенциал как раз таки наследуется, на отдельных контролируемых базах их можно обучать по специальной программе воспитывать так же по особой программе, так чтобы они получали нужную долю любви и заботы, чтобы у них формировалось привязанность. И не возникало проблем по поводу того что их воспитывали на военной базе например. Преподавателей и воспитателей можно подобрать специальных по исключительному отбору со всей страны. Так ещё раз повторяю почему никто не занимается подобным?
Аноним 13/01/21 Срд 13:31:26 526362209
index.jpeg 5Кб, 225x225
225x225
>>526361
>Почему никто не пытается ввести сверхумных людей где-нибудь на отдаленной базе, чтобы потом из воспитать для продвижения прогресса?
Потому что невозможно контролировать интеллект, который тебя превосходит.
Ты слишком глупый, чтобы понять это, поэтому у тебя такие вопросы детские и возникают.
Аноним 13/01/21 Срд 13:37:36 526363210
>>526361
>Почему никто нормально не вкладывается в евгенику?
Потому что ее скомпрометировали одни ребята в серой форме, а современная левацкая повесточка вообще исключает возможность существования евгеник, хуегеник, да даже с антропологией всё плохо.
>почему никто не пытается ввести сверхумных людей где-нибудь на отдаленной базе, чтобы потом из воспитать для продвижения прогресса?
Где-то в поднебесной мб и пытаются, но нам об этом мало что известно. Ты предлагаешь легализовать подобные эксперименты? Ты сам готов был бы родиться в таком лагере, и если что, пойти в утиль?

Или ты предлагаешь просто воспитывать потенциально вумных детишек отдельно? Так для того и существуют разного рода конкурсы, олимпиады, гимназии и просто. А ебать умных с умными по принуждению, ну хуй знает. Вряд ли они будут рады. Да и кому столько умников нужно? Вот серьезно. Ты думаешь власть это про ум? Отнюдь.
Аноним 13/01/21 Срд 14:04:31 526364211
>>526362
На самом деле, невозможно нихуя контролировать, потому что принципиальное существование "контроля", подразумевает принципиальную возможность существования "управления", которого не существует, потому что мы биороботы, и срабатываем однозначно предопределённым образом, на принципах детерминизма. Да, мы можем испытывать иллюзию управления, и какого-то контроля, но только в рамках нашего манямирка, никоим образом не отражающего всю объективную реальность.
Аноним 13/01/21 Срд 14:13:46 526365212
>>526361
>Почему никто нормально не вкладывается в евгенику?
Почему это? Вкладываются.
Правители государств забирают деньги у опущенных рабочих шлюх налогами, и отдают их всяким дегенератам в виде пособий, льгот и бесплатной социалки, чтобы дегенераты больше плодились, проводя тем самым отрицательную селекцию. Иногда даже мюслей чурок и нигров завозят на пособия от рабочих шлюх, чтобы ещё ускорить и усилить отрицательный отбор.
А ты думал евгеника только положительная может быть?
Нет, евгеника это просто общий термин, самого факта наличия и проведения отбора.
Аноним 13/01/21 Срд 14:15:27 526366213
>>526363
>Потому что ее скомпрометировали одни ребята в серой форме, а современная левацкая повесточка вообще исключает возможность существования евгеник, хуегеник, да даже с антропологией всё плохо.
А, и да, положительную евгенику не проводят в основном из-за пидарасов евреев, которые проталкивают везде где могут отрицательную селекцию дегенератов, евреи таким способом геноцидят население европы и америки, чтобы убрать конкурентов.
Аноним 13/01/21 Срд 14:17:26 526367214
>>526363
>Ты предлагаешь легализовать подобные эксперименты?
Так они уже легализованы, лол, только в отрицательном направлении.
А чтобы "легализовать" в положительном-никого в утиль пускать не надо, надо только отменить по максимуму налоги и бесплатную социалку, чтобы дегенераты сами умирали с голода.
Или ты считаешь что не содержать дегенератов за свой счёт это тоже плохо?
Аноним 13/01/21 Срд 14:21:22 526368215
>>526360
Слишком много объяснять, явление довольно сложное. Особенно формализм энергии, движения и квантовых хуйни.
Короче. Магниты магнитят из того, что электроны не только являются заряженными частицами, да еще маленькая магнитами. По себе они ориентированы произвольно, но при определенных условиях магнитам из электронов становится выгодно синхронизироваться. Эту хуйню еще называют спонтанной нарушение симметрии. Еще можно проиллюстрировать на примере образования кристаллов. Кристаллы это упорядоченные структуры, где есть выделенные направления. Жидкости или газы не обладают такими свойствами. Однако при кристаллизации газов или жидкостей, выделяется одно направление и экспоненциально усиливается, образуя кристалл.
Аноним 13/01/21 Срд 14:35:42 526369216
>>526361
>Почему никто нормально не вкладывается в евгенику?
Потому что в своем начале оказалась зашкваренна мистической хуитой.
А с учетом развитии нейронаук и генетики, оказалось, что умность людей является по сути дикой мешаниной из рандомных факторов, многие из которых не очень генетические. Вычленить что-то осмысленное из этого пазла пока что не удается.
А если мы будет отбирать людей на умность, то выйдет отбор по рандомному призраку, слабо связанным с самой умностью.
Кстати наши предки отбирались не по умности, а по степени социализации и конформности. По тем при знакам, что сейчас не считается особым признаком умности и интеллекта.
Аноним 13/01/21 Срд 14:47:45 526370217
>>526368
>Магниты магнитят из того, что электроны не только являются заряженными частицами, да еще маленькая магнитами.
А как же токи, циркулирующие?
Вопрос - откуда у электронов энергия на движение по-прежнему остался без ответа.
Аноним 13/01/21 Срд 14:59:01 526371218
>>526370
Тут не совсем осмыслено разбираться в циркуляции токов для квантовых явлений. Все сводиться к ориентации спинов, епта.
А электрон не может существовать без внутреннего "вращения", которое на самом деле не механическое вращение.
И опять очень много надо объяснять о формализме энергии, движения и квантов.
Аноним 13/01/21 Срд 15:29:16 526373219
>>526369
>что умность людей является по сути дикой мешаниной из рандомных факторов, многие из которых не очень генетические
Забавно что это оказалось после того, как евреи сказали "учёным" так считать.
Аноним 13/01/21 Срд 15:31:58 526374220
>>526371
>надо объяснять
Не можешь объяснить - хотя бы скажи где уже объяснено, чтобы прочитать.
Аноним 13/01/21 Срд 15:54:48 526377221
>>526374
Про спин электрона курс квантовой механики. Если не можешь в него, иди на постнауку или на ютуб. Сейчас дохуя материала на разные уровни.
Спонтанное нарушение симметрии много где появляется, но обычно это квантовая теория поля, хотя по идеи это должна быть статистическая физика и механика сплошных сред. Тут сам ищи.
Аноним 13/01/21 Срд 16:12:27 526378222
>>526367
>А чтобы "легализовать" в положительном-никого в утиль пускать не надо, надо только отменить по максимуму налоги и бесплатную социалку, чтобы дегенераты сами умирали с голода.
Когда будешь выдвигаться в президенты, маякни, я откопаюсь из подушек, слезу с дивана и наконец проголосую хоть за кого-то.
>Или ты считаешь что не содержать дегенератов за свой счёт это тоже плохо?
Не, я как раз за то, чтобы поддерживать только тех, кто реально полезен. И матери-одиночки, нигры и прочие в категорию "реально полезных" не входят. Но это оффтоп, мы всё-таки про евгенику скорее.
Аноним 13/01/21 Срд 16:20:50 526379223
>>526364
Хуйню несешь. Свободная воля человека обеспечивается квантовой физикой, которая делает мир в принципе непредсказуемым. Никто и никогда не сможет предсказать на 100% поведение человека. Поэтому просто вздорно в 2021 году, с учетом всего, что мы знаем сегодня о квантовой физике, говорить о каком-то детерминизме. У тебя представления о мире времен Лапласа.
Аноним 13/01/21 Срд 16:27:07 526380224
>>526379
>Свободная воля человека обеспечивается квантовой физикой, которая делает мир в принципе непредсказуемым.
Не определяется, иначе вы бы швыряя шайбу натрия в воду, получали бы не бубух и огонь, а нечто совершенно новое. Квантовая физика хороша в определенных системах. Не стоит ее примерять на всё, что уже есть.
>Никто и никогда не сможет предсказать на 100% поведение человека
Безусловно.
>Поэтому просто вздорно в 2021 году, с учетом всего, что мы знаем сегодня о квантовой физике, говорить о каком-то детерминизме.
Не вздорно, не стоит примерять квантовую физику на всё, что есть. Квантовая физика на данный момент основа для всего. Далее надстраивается химия, поверх химии надстраивается биология. Не нужно разрывать эту цепочку. Иначе херня получится.

Мимодругой анон
Аноним 13/01/21 Срд 17:46:32 526386225
>>526379
> Свободная воля человека обеспечивается квантовой физикой
Случайные события не дают свободы воли.
Они дают только случайность. Чтобы была "свобода воли" должна быть воля, сознания, а говоря что "сознание" человека-квантовое, ты сам заявляешь что никакого сознания и воли нет, есть только набор случайных событий.
Аноним 13/01/21 Срд 17:51:49 526387226
>>526379
>Поэтому просто вздорно в 2021 году, с учетом всего, что мы знаем
Да нихера не знают, до сих пор спорят, доказательств внятных отсутствия детерминизма нет, все которые есть-больше похожи на всякие маняфантазии и мысленные эксперименты опять же-таки с кучей маняфантазий, условностей и нерассматривающие выход за условия этого мысленного эксперимента.
Аноним 13/01/21 Срд 21:21:28 526401227
>>526379
>Свободная воля человека обеспечивается квантовой физикой, которая делает мир в принципе непредсказуемым.

Ну что, много свободной воли с помощью монетки сделал? Ты ебанутый, ты сам себе противоречишь. Свободная воля невозможна ни при случайных событиях, ни при предопределенных, ни при любой комбинации.

И это я еще не говорю о том, что квантовая механика случайна только в парочке интерпретаций, а в общем случае волновая функция меняется непрерывно по конкретному закону.
Аноним 13/01/21 Срд 21:37:11 526403228
>>526380
>>526386
>>526387
https://youtu.be/trJdvW9s0K0 https://m.lenta.ru/news/2016/04/09/theorem/ Просто оставлю это здесь. Ну и еще добавлю, что для начала вообще нужно определиться с тем, что мы понимаем под свободной волей. Я понимаю под этим ситуацию, когда есть некий субъект, есть как минимум два варианта и есть безусловная возможность выбора у этого субъекта между имеющимися вариантами. Безусловная возможность выбора - это невозможность со стопроцентной точностью предсказать выбор данного субъекта, то есть он реально может выбрать либо первый, либо второй вариант, и мы не знаем, что именно он выберет - мы можем лишь просчитать вероятность. Это называется свободный выбор. Свободный выбор - это и есть свободная воля. В видео объясняется как раз, каким именно образом квантовая физика связана со свободной волей, с головным мозгом. Мозг - физическая система, а любая физическая система очень чувствительна к начальным условиям. Малейшие изменения в этих условиях приводят к неизбежному изменению поведения этой системы. Каждое мгновение в нашем мозге происходят квантовые события, которые влияют на его поведение как физической системы в целом. Поскольку квантовые события являются истинно случайными и поскольку мы, наше поведение - это наш мозг, наш выбор в той или иной ситуации нельзя предсказать, и, следовательно, у нас есть свободная воля в том значении этого термина, о котором я написал. Ну и про то, что квантовая физика действительно имеет случайную природу, что нет никаких скрытых параметров, рекомендую прочитать книгу швейцарского физика Николя Жизана "Квантовая случайность". Проводили массу исследований этого вопроса на протяжении долгого времени, и в результате не нашли никаких следов скрытых параметров - истинно случайный характер квантовых явлений был подтвержден с очень высокой степенью точности.
Аноним 13/01/21 Срд 22:09:44 526404229
>>526403
Ясно. Не пиши сюда больше, мне стыдно.

P.S. Научпоп в жопу засунь, иди читай нормальные книги.
P.S.S. У тебя любой фотон обладает свободой волей, куку-епта.
Аноним 13/01/21 Срд 22:31:06 526406230
>>526403
>Безусловная возможность выбора - это невозможность со стопроцентной точностью предсказать выбор данного субъекта, то есть он реально может выбрать либо первый, либо второй вариант, и мы не знаем, что именно он выберет - мы можем лишь просчитать вероятность.

Если ТЫ не видишь закономерности (процесса выбора), то это не значит что её нет.
А если она, эта закономерность есть, то у процесса (процесса выбора), во времени, есть причина, закономерность и следствие.
И всё это значит, что зная закономерность и причину (состояние предшествующее выбору),
можно высокоточно смоделировать заранее, а значи и предсказать следствие (результат выбора).
Заметь, что закономерность, связывает причину и следствие, разбросанные во ВРЕМЕНИ, и что процесс протекает во ВРЕМЕНИ,
то есть принцип причинности, связан с временным измерением, от которого кванты никуда не денутся.
Наверняка, поэтому, унитарная эволюция, в квантовой механике детерминистична,
хотя вот декогеренцию, вроде как - хуй смоделируешь.
Аноним 13/01/21 Срд 22:34:51 526407231
Аноним 13/01/21 Срд 22:40:23 526408232
>>526404
Ебать ты жидко обосрался, чушок. Ничего по существу не написал, одни сопли и слюни. Понимаю.
Аноним 13/01/21 Срд 22:52:16 526409233
>>526408
По твоему мнению, если вместо принятия решений просто подкидывать монету, то это будет свободой воли. Ясно.
Аноним 13/01/21 Срд 23:04:50 526410234
>>526409
Чмо тупорогое, подбрасывание монетки - это псевдослучайный процесс, а не истинно случайный. Так что ты поел говна со своими всратыми аналогиями.
Аноним 13/01/21 Срд 23:24:53 526411235
>>526410
Но ведь из-за детерминистичности унитарной эволюции, квантовые мельтешения тоже не случайны.

Истинно-случайные явления - спонтанные явления, такое было только одно - появление планковской эпохи. Почему оно спонтанно? Да потому что оно беспричинно, как-бы, и не было другой планковской эпохи до самой первой планковской эпохи, а уже после, было.
Разве есть что-то реально спонтанное во вселенной? Сам подумай.
Вот что-то произошло. А почему это произошло, в чём ПРИЧИНА? Причина, в состоянии, предшествующем текущему состоянию. А что было до текущего состояния, на планковский отрезок? А было вот такое состояние. И так далее, для любого кванта.
Спонтанное же явление - явление, которое не имеет причины, а значит не имеет состояния, предшествующего его появлению. Или что-то такое, что не должно быть вообще, исходя из всех состояний. Что-то неведомое, из ниоткуда берущееся. Нечто подобное, разве, реально, блядь, есть?

Мимо этот >>526406
Аноним 14/01/21 Чтв 00:03:08 526415236
>>526410
Измеряй спин электрона, один хуй это не будет свободной волей
Аноним 14/01/21 Чтв 03:26:21 526420237
>>526410
>подбрасывание монетки - это псевдослучайный процесс, а не истинно случайный
Так если существуют истинно случайные процессы, то они вносят изменения и в поведения монеты при подбрасывании, и делают его тоже истинно случайным.
Ебать ты обосрался даунич.
Аноним 14/01/21 Чтв 04:50:33 526422238
>>526403
>https://youtu.be/trJdvW9s0K0
Это даже не смотрел, потому что переведенный зарубежный научпоп, и ссылаться на подобное смысла особого нет
https://m.lenta.ru/news/2016/04/09/theorem/
Это прочитал, только оттуда не следует, что свобода воли отсутствует, потому что квантовая физика. А ты это нам тут и пытаешься доказать.

Знаешь, ты мог привести эксперименты Либетта в качестве аргумента, что-нибудь из Science или Nature по теме. Но ты пытаешься нам тут доказать, что поведение то, чего мы называем личностью, сознанием, квалией какой или еще, ведет себя рандомно, потому что квантовая физика блять. Ты серьезно идиот или прикидываешься?
Аноним 14/01/21 Чтв 05:08:37 526423239
>>526422
А он разве не опровергнуть это пытался?
Аноним 14/01/21 Чтв 05:22:02 526424240
>>526423
Да, но товарищ мог бы хоть что-то из областей подтянуть, где понятием свободы воли оперируют, извратить, переврать, но нет, он тащит квантовую физику. Это абсурдно.
Аноним 14/01/21 Чтв 05:28:39 526425241
>>526424
Самоцитирую себя
>он тащит квантовую физику. Это абсурдно.
Ему с такой линией доказательства, нужно увязывать вообще всю цепочку, начиная от квантовой физики, переходя в обычную, мигрируя через химию, потом нырять в биологию, а оттуда бог весть знает еще куда нырять, ведь даже в таких вещах как вопрос свободы воли, и сознания, которое эту саму свободу воли может реализовать, еще разобрались.

Но нет, ему похуй, он даже не пытается, он просто скачет из квантовой физики прямиком в нейрофизиологию, или где там свободу воли изучают, барон мюнхгаузен просто.
Аноним 14/01/21 Чтв 05:29:43 526426242
>>526425
> еще разобрались.
Корректировка, еще не разобрались до конца.
Аноним 14/01/21 Чтв 11:46:52 526447243
>>526403
Пожалуй, разберу эту вкуснятину.
>для начала вообще нужно определиться с тем, что мы понимаем под свободной волей.
Именно. Нужно дать чёткое определение, и оно как-бы есть уже.
>https://ru.wikipedia.org/wiki/Свобода_воли
>Свобода воли (греч. τὸ αὐτεξούσιον или τὸ ἐφ᾽ ἡμῖν; лат. liberum arbitrium) — это возможность человека делать выбор вне зависимости от обстоятельств.
>Под обстоятельствами здесь могут пониматься как чисто внешние обстоятельства
>(насилие, принуждение, вознаграждение и т. д.), так и внутренние (инстинкты, симпатии и антипатии, предубеждения и т. п.).
То есть, только к людям относится, это определение, по определению.

Ты предлагаешь альтернативное определение, пошире, начиная с самых низов - с определения "свободы выбора".
Уже хорошо.

>Я понимаю под этим ситуацию, когда есть некий субъект...
В том числе и животное, организм, или, скажем - иноформационная система, верно?
Сразу выделю ключевое слово - субъект.
Если это определение истинно, то и наличие субъекта - тоже должно быть истинно,
в реальности, которая может быть объектвной, то есть являющейся - сплошным объектом.
Дальше...
>есть как минимум два варианта и есть безусловная возможность выбора у этого субъекта между имеющимися вариантами.
Под это определение попадает любая многозначная функция: https://ru.wikipedia.org/wiki/Многозначная_функция

Теперь полностью, цитата:
>Я понимаю под этим ситуацию, когда есть некий субъект,
>есть как минимум два варианта и есть безусловная возможность выбора у этого субъекта между имеющимися вариантами.
А если это не субъект, а объект осуществляет выбор, скажем, вероятностная нейронная сеть,
исполняет некий - https://ru.wikipedia.org/wiki/Вероятностный_алгоритм ?
Будет ли её выбор (определенение параметра многозначной функции) - свободным, в таком случае?

Дальше...
>Безусловная возможность выбора - это невозможность со стопроцентной точностью предсказать выбор данного субъекта,
>то есть он реально может выбрать либо первый,
>либо второй вариант, и мы не знаем, что именно он выберет - мы можем лишь просчитать вероятность.
Выше уже сказали, что отсутствие знаний закономерности, не означает отсутствие этой закономерности,
а значит отсутствие принципиальной возможности, смоделировать процесс выбора, с учётом всех возможных параметров.
>Это называется свободный выбор.
То есть, как я понял, ключевым компонентом является именно невозможность определить результат выбора,
то есть сокрытие закономерности процесса выбора, позволяющей предсказать этот выбор?
Если закономерность сокрыта, и выбор не может быть предсказан, то выбор свободный, иначе - не свободный. Так ведь?

Дальше...
>Свободный выбор - это и есть свободная воля.
Ну тут ты уже вообще обобщаешь.
Термин "свобода воли", тесно связано с термином "интеллект", и исходя из вышеописанного определения,
это некая возможность (принципиально не исключённая возможность, разумеется),
возможность делать выбор, независимо от ОБСТОЯТЕЛЬСТВ,
коими являются причины, инициализирующие закономерный процесс выбора, в соответствии с закономерностью этой,
и ведущей этот процесс в конкретный, определённый этими параметрами результат (в результат выбора).
То есть, вот, блядь, как сказать... Есть значит причины (обстоятельства), а есть их отсутствие, понимаешь.
Представь себе, что интеллект, который рассматривает некую систему, видит БАГ В СИСТЕМЕ, и ХОЧЕТ его исправить,
но не может, попросту потому что НЕТ ПРИЧИН для инициализации некоего процесса исправления.
Ну нет их, и всё тут. Не инициализируется процесс исправления, этими вот, ебучими, причинами-обстоятельствами.
Но интеллект, находит эти причины, рождает их, или хуй знает откуда - из воздуха, из квантов берёт,
из говна и палок делает и использует их как причины, и - ДЕЛАЕТ ЭТО,
он инициализирует процесс ИСПРАВЛЕНИЯ БАГА в системе,
и таким вот образом - производит УПРАВЛЕНИЕ, ВОЛЕВОЕ УПРАВЛЕНИЕ, свободной волей своей.
Почему? Да просто потому что так захотелось, и нет никаких объективных причин для этого. Вот!
И теперь сразу, сходу - вопрос, а возможно ли в принципе, подобное? Да/нет.
Возможно ли, существование такой вот, свободной воли,
или всё же, каждое действие, каждый процесс, определен именно причинами,
то есть обстоятельствами, или состояниями, предшествующими ВО ВРЕМЕНИ, инициализации процесса,
и главное - ИМЕЮЩИМИ МЕСТО БЫТЬ, и не имеющими место НЕ БЫТЬ?

Аноним 14/01/21 Чтв 11:47:18 526448244
>>526403
Продолжу >>526447
Дальше...
>В видео объясняется как раз, каким именно образом квантовая физика связана со свободной волей, с головным мозгом.
Вообще-то мозг, это довольно сложная система, да. 100 миллиардов нейронов где-то по 10000 связей на каждый,
и один нейрон может возбудиться хуй знает от чего, и совозбудить кучу нейронных сетей, да.
Но могут ли квантовые события из микромира, действительно влиять,
на такие макроскопические по сравнению с ними объекты, как клетки, как нейроны?
Просто представь, что в клетках мозга, происходит, радиоактивный распад какого-нибудь природного изотопа - Калия-40, например.
Квантовое явление, как-бы, да, но а что толку от этого?
Продукты распада настолько маленькие, и настолько быстро вылетают осколки распада,
что они не могут задеть нейроны и возбудить их...

>Мозг - физическая система, а любая физическая система очень чувствительна к начальным условиям.
>Малейшие изменения в этих условиях приводят к неизбежному изменению поведения этой системы.
>Каждое мгновение в нашем мозге происходят квантовые события,
>которые влияют на его поведение как физической системы в целом.
Вот здесь, довольно интересно. Ты предлагаешь рассмотреть мозг, как хаотическую систему,
где малейшее отклонение, вызывает изменение системы, по принципу "эффекта Бабочки".
Но, что если такая система, не в мозгу, а где-нибудь, хуй знает где, на компе скажем, работает.
Будет ли такая система "субъектом", со "свободной волей"?
Тащемта, система, она и в Африке система, и её поведение - можно смоделировать,
на принципах той же теории систем и системного анализа.
Однако, ты говоришь, что хаотическая система, не может быть принципиально смоделирована,
из-за того, что кванты изменяют её поведение, а кванты мол - хуй смоделируешь.
Заебись как гонче получается, но получается это для любых систем, не обязательно для каких-то "субъектов".

Дальше...
Представим себе, процесс выбора, некоей хаотичной системой,
это процесс, у него есть причина, закономерность, и следствие.
Пусть закономерность будет реорганизующаяся, и пусть её реорганизуют,
по принципу эффекта Бабочки - рандомно-мельтешащие, и немоделируемые - кванты.
Такая система, как-бы, сможет делать "свободный выбор", но у неё не будет "свободы воли",
потому что рандомные мельтешения квантов, нихуя не конструктивны, а вероятностны и случайны, это рандом блядь.
Чтобы все эти мельтешения и выборы, укладывалось в "свободную волю",
система должна иметь некую "алгоритмику организации этих мельтешений",
как алгоритмика психики в мозгу человека.
Однако, наличие этой алгоритмики, делает процесс организации квантовых мельтешений,
а следовательно осознанный выбор - предсказуемым, закономерным, и моделируемым теми же вероятностными алгоритмами,
не смотря на то, что сами мельтешения, и выборы эти "какбэ-свободные".

Вообще, если так удуматься, то почему это вдруг нельзя смоделировать кванты?
Кванты, они обладают некими "степенями свободы", что в данном контексте,
как-бы намекает на свободный выбор и свободу воли,
но, степени свободы, это просто координаты в различных измерениях,
где частица может занять "произвольное" положение, и не более того.
То есть, блядь, это что ж получается - они, эти кванты, не "могут" нихуя выбирать сами, где им быть и как им срать,
они могут разве что положение занять, какое-то, в сетке координат,
которое обусловлено, разумеется - обстоятельствами, причинами, то есть предыдущим их состоянием.
Ну а поскольку кванты эти, во времени спинами своими мельтешат,
то каждому состоянию предшествует предыдущее состояние,
которое однозначно определяет текущее состояние.
Значит, зная изначальное состояние и закономерность его преобразования в текущее состояние
(то есть полное описание квантовомеханической системы), таки-можно смоделировать кванты,
а зная изначальные параметры хаотичной системы, на которую воздействуют кванты, можно смоделировать и рандом и выборы.

Получается, что рандома, и спонтанности - в принципе нет, блядь, если всю хуйню так можно смоделировать.
Аноним 14/01/21 Чтв 12:12:03 526449245
>>526377
>Спонтанное нарушение симметрии
Звучит как "воля Божья".
Я спрашивал не про неё, а откуда в закрытой системе типо магнит берется энергия на поддержание циркулирующих токов.
Я не пытаюсь доебаться до тебя.
Но ты не ответил на мой вопрос, просто сказав ищи сам.
При этом смешав в кучу разные области в которых якобы надо искать.
Если ты не можешь объяснить, то так и скажи.
В незнании нет ничего плохого для того, кто тянется к знаниям.
Имхо, если бы ты сам знал и понимал, как это работает, то мог бы объяснить.
С другой стороны, если бы ты знал это, то владел бы способом извлекать энергию прямо из пространства.
Аноним 14/01/21 Чтв 13:40:07 526454246
>>526449
>системе типо магнит берется энергия на поддержание циркулирующих токов.
Отсутствие потерь=вечная циркуляция.

А она вообще там есть, эта циркуляция токов?
Аноним 14/01/21 Чтв 14:01:57 526459247
>>526449
Ну бля. Для обычного случая потери энергия происходит из диссипации из-за сопротивления/трения считай. Если убрать его, будет тебя постоянное магнитное поле.
Трение является коллективным явлениям и на микромаштабах пропадает.
В постоянном магните вообще нет циркулирующих токов и каких либо движущихся зарядов, магнитное поле создается электронами, которые решили "повернуться" в одну сторону.
Сами электроны являются маленькими магнитами, магнитное поле которых порождается его волновыми свойствами и не связанно с механическим движением, которому можно приписать энергию.
Аноним 14/01/21 Чтв 14:01:57 526460248
>>526449
>С другой стороны, если бы ты знал это, то владел бы способом извлекать энергию прямо из пространства.
Я не знаю к чему ты это написал. Но любой пятиклассник может извлекать энергию из магнита. Магнит от такого насилия конечно же размагнитится. При этом в магнитах из редкоземельных металлов, никаких циркулирующих токов нет. Ибо квантовая электродинамика отличается от обычной, и там есть возможность (спин) иметь магнитное поле без вращения.

Аноним 14/01/21 Чтв 14:32:36 526465249
>>526459
>которые решили "повернуться" в одну сторону.
А как они могли повернуться в одну сторону, если у электронов вообще орбиты нет, не то что круговой/плоской(а не "шаровой"), а только поле вероятности, и тоже не круговое, а "шаровое".
А шар он не плоская орбита, он со всех сторон одинаковый.
Аноним 14/01/21 Чтв 14:52:20 526469250
>>526465
Так они не орбитальным моментом поворачиваются а спином.
Аноним 14/01/21 Чтв 15:00:49 526471251
>>526469
А спин он типа всегда в одном положении остаётся, несмотря в каком месте и с какой стороны поля вероятности электрон появился?
Забавно.

Ну тут явно чувствуется костыль и маняфантазия, придумывания чтобы оправдать тянущиеся цепочкой один за одним вопросы возникающие.(не твои ответы тут на мои и другого анона вопросы, а учёных придумывания, на возникающие о теории вопросы.)

В РЕАЛЬНОСТИ ™ оно явно всё не так... без таких костылей с придумываниями и додумываниями отговорок типа "а вот где бы он не появился в поле вероятности с какой стороны, спин всегда в одном положении получается (относительно чего, лол? и как он ЗНАЕТ в каком положении ему надо появляется)
Аноним 14/01/21 Чтв 15:35:01 526476252
>>526471
>>526471
Странная фраза "появляется в поле вероятности". Ты будто как-то неправильно воспринимаешь КМ.

А так, вообще, волновая функция в обычном КМ (нерелятивистском) состоит из двух компонентов (отдельно для спина вверх отдельно для спина вниз).

Ну и тебе надо понять, что спин хоть и векторная (ребят кто понимает, не ругайте за слово векторная, знаю что спинор) величина, но всё же квантовая, и там такие вещи возникают, о которых думаешь ночами первую неделю изучения и такой "да блять, как так".
Аноним 14/01/21 Чтв 15:38:29 526477253
>>526465
Все не так.
Электрон обладает внутренними (квантовыми) параметрами. Как система он эволюционирует только по внутренним параметрам. Когда происходит измерение, мы на мгновение вторгаемся в этот мир и успеваем подсмотреть фрагмент что там происходит. Подсмотренное преобразуется во внешние параметры вроде импульса и координат. Нельзя просто связать и объединить внешний и внутренний мир. Для этого нужно вводить специальные преобразования - этакий мост между мирами.
Волновая функция является чисто внутренним параметром, ее нельзя задать во внешнем мире как поле вероятностей без последствий.
Спин это тоже внутреннее свойство и его нельзя отождествить с каким либо внешним параметром. Однако спин может проявить себя на результаты измерения, причем довольно замысловатым образом. Частное проявление этого является появление магнитного момента. Про поворот этого магнитного момента и говорилось.
В сложных системах иногда возникают ситуации, когда магнитные моменты синхронизируются. Это и есть смотрение в одну сторону.
Аноним 14/01/21 Чтв 15:39:27 526478254
>>526476
>там такие вещи возникают, о которых думаешь ночами первую неделю изучения и такой "да блять, как так".
Но возникают же "не там" а в костыльной фантазии в попытке как-то работающую модел сделать.
Аноним 14/01/21 Чтв 15:54:21 526485255
>>526476
Ты путаешь.
Нерелятивистском случае вообще нет спина, если его специально самим не вводить как отдельный.
Спин сам по себе появляется в релятивистском случае, потому что там волновая функция вообще говоря является четырьмя двумя комплексными числами и она разделяется на два диаметрально противоположных случая.
Аноним 14/01/21 Чтв 16:00:03 526487256
>>526478
А кроме моделей реальности не существует. Ведь даже мир вокруг мы воспринимаем через органы чувств и обработки мозгом т.е. модель.
Аноним 14/01/21 Чтв 16:06:57 526488257
>>526485
>Нерелятивистском случае вообще нет спина, если его специально самим не вводить как отдельный.
В нерелятивистском случае спин не входит в гамильтониан, но без него ты не соберешь волновую функцию многоэлектронной системы (ибо антисимметричность никто не отменял). Поэтому даже в нерелятивистском случае помимо трех координат вводят спиновую координату, ну либо разбивают волновую функцию на две компоненты, либо (так как гамильтониан не включает в себя спин), описывают волновую функцию как произведение координатной части и спиновой. Попытка описать атом лития без спина, приведет тебя к фейлу например.

В релятивистском да, 4 компоненты, матрицы Дирака и вся хуйня...
Аноним 14/01/21 Чтв 16:41:02 526489258
>>526487
>А кроме моделей реальности не существует.
Конечно существует шиз, ведь придумывают постоянно новые и новые модели.
Аноним 14/01/21 Чтв 17:03:26 526494259
>>525425 (OP)
Почему атом до сих пор называют атомом (от др.-греч. ἄτομος «неделимый, не разрезаемый»), если он делится и разрезается?
Аноним 14/01/21 Чтв 17:14:44 526495260
Аноним 14/01/21 Чтв 17:53:27 526502261
>>526495
А когда разделив "атом" найдут некоторую хрень, которая действительно не делится и не режется, как её назовут?
Аноним 14/01/21 Чтв 17:55:35 526503262
Аноним 14/01/21 Чтв 18:04:04 526506263
Аноним 14/01/21 Чтв 18:06:31 526507264
>>526506
Потому что атом состоит из ядра и электронов. Ядро из протонов и нейтронов блять. Я так понимаю учитывая контекст вопроса
>А когда разделив "атом" найдут некоторую хрень, которая действительно не делится и не режется, как её назовут?
Про стандартную модель мне даже говорить нет смысла
Аноним 14/01/21 Чтв 18:10:59 526511265
>>526507
>стандартную модель
Это в которой всякие квази-частицы? Ну их будут делить ещё дальше, а потом как-нибудь окажется что дальше делить нельзя. Всё что найдут будет неделимое и неразрезаемое
Аноним 14/01/21 Чтв 18:17:54 526512266
>>526511
Чтобы дальше после сегодняшнего уровня разбить частицы нужен коллайдер размером больше планеты, при этом непонятно как такую маленькую хуйню детектить, уже давно детектят по очень и очень косвенным признакам, интерпретировать которые можно сотнями фантазий.

При этом о неделимости дальнейшей никто не говорит.
Если всё окажется волновым-это делимое или не делимое? Возмущение моле может быть сколь угодно малым потенциально.
Можешь сразу придумать себе ограничение неделимости-размер длины волны движущейся со скоростью света, с такой частотой, с которой она будет меньше размера Шварцшильда для данной энергии волны с такой частотой.
Аноним 14/01/21 Чтв 18:29:36 526513267
>>526512
Древние греки как-то ведь дошли до идеи атомов задолго до того, прежде чем появился коллайдер. Собственно вся современная наука выросла из этой идеи атомарности материи. Хочешь сказать, что предела деления нет?

>Чтобы дальше после сегодняшнего уровня разбить частицы нужен коллайдер размером больше планеты
Ну это всё не более чем временные технические затруднения. Не вижу причин, чтобы когда-то они не порешались
Аноним 14/01/21 Чтв 19:39:51 526520268
>>526513
Читай полностью, рабзить не основная проблема, предел тоже тебе вон дан.
Аноним 14/01/21 Чтв 21:01:39 526532269
>>526520
>размер длины волны движущейся со скоростью света, с такой частотой, с которой она будет меньше размера Шварцшильда для данной энергии волны с такой частотой
Что-то сложна, но цифры-то все конечные. Ладно была бы где-то бесконечность, а так значит предел принципиально не недостижим
Аноним 14/01/21 Чтв 21:06:25 526533270
>>526511
Квази, штази. Бозон же подтвердили при помощи Ядрённого Коллайдера. Отклонений от стандартной модели пока найти не удалось. У тебя 4 часами ранее атом не делился, а это уже заявочка бро. Хочешь делить бозоны и нейтрины, добро пожаловать. Хочешь доебываться со своим ножом до физиков, литр бозонов спермы на ебало и пишов нахуй ваще.
Аноним 14/01/21 Чтв 21:14:39 526534271
>>526511
>Всё что найдут будет неделимое и неразрезаемое
Физкульт-привет от алхимиков, геоцентристов и преследователей атомистов. Вот вроде 2000 лет прошло, а нихуя не меняется. Если тебе хочется порассуждать о том, что куча народу (нуууу тупыыыыееее) всё чет делит и делит, а никак поделить не может. Пиздуй в тред к философам, это вообще не здеся, и ни разу не туточки. SCI это про теории которых придерживаются и про практику. А не про ля тупые, кварки ножом не режутся, чо дальше будете делать ГЫГЫГЫ лалки. Иди терраформированием ануса соседской ЕОТ занимайся, или ещё чем.
Аноним 15/01/21 Птн 08:26:42 526555272
16104365506360.jpeg 15Кб, 637x532
637x532
Поскольку в спэйсаче мне не ответили, то спрошу тут.
Общеизвестно, что существует эффект пращи.
На бумаге он выглядит как-то так.
Вопрос первый, будет ли эффект работать, если заменить гравитационное взаимодействие не магнитное?
Второй, чем объясняет современная наука процесс передачи энергии в случае гравитации?
Третий - если будет работать с магнитным взаимодействием, то используется ли это где-то в технике?
Аноним 15/01/21 Птн 09:26:05 526557273
>>526555
Если ты про гравитационный маневр то на первый взгляд должно работать и с магнитным полем, но обрати внимание что для гравитационного маневра нужно три тела. К примеру в случае маневра вокруг планеты очень важно что эта планета вращается вокруг звезды (собственно импульс получаемый апаратом в результате маневра и берется из импульса планеты которая вращается вокруг звезды). Гравитационный маневр с двумя телами в системе не возможен.
Если ты про хуйню с двойными звездами то там все завязано на свойствах гравитации (связь с массой) и с магнитным полем ты такого не получишь. Конечно можно симулировать что-то подобное если у тебя есть возможность изменять параметры мегнитного поля.
Аноним 15/01/21 Птн 09:54:57 526558274
>>526557
>импульс получаемый апаратом в результате маневра и берется из импульса планеты которая вращается вокруг звезды
А если планета улетела от звезды и просто движется в межзвездном пространстве со своей скоростью, то маневр не удастся? Почему?
Аноним 15/01/21 Птн 10:11:28 526559275
>>526558
Да. Вся суть в маневра в том, что есть связанная гравитационная система из двух тел и за счет своей связи они могут совершить работу над третьем.
Если попробовать с одиночным системой, то либо они столкнуться, либо разлетятся в разные стороны - выходит нечто похоже на упругое столкновение.
Аноним 15/01/21 Птн 10:14:49 526560276
>>526013
Спустя почти неделю экспериментов, на столе фрукты стали портиться заметно медленнее при тех же условиях - часть фруктов в пакете, часть фруктов мытые в глубокой салатнице из икеа. Блин, я даже не знаю, на подоконнике в целом прохладнее и продувается, почему фрукты портятся там быстрее.
Осталось провести side to side эксперимент.
>>526014
Чего ты подкалываешь.
Аноним 15/01/21 Птн 10:19:17 526561277
>>526559
>и за счет своей связи они могут совершить работу над третьем.
Откуда ты это взял?

https://www.youtube.com/watch?v=1s6_4qX-u2o

Где в системе уравнений присутствует Солнце?
Аноним 15/01/21 Птн 10:22:24 526562278
>>526560
>Чего ты подкалываешь.
Героин. Я серьезно тащемта говорил.
Аноним 15/01/21 Птн 10:43:51 526566279
>>526561
Лол, она в сам начале говорит о условии сохранения угловой/орбитальной скорости. т.е относительно Солнца.
Аноним 15/01/21 Птн 10:55:03 526567280
>>526566
>т.е относительно Солнца.
Почему не относительно центра Галактики, например.
Аноним 15/01/21 Птн 11:04:01 526568281
>>526567
По условию задачи у нас Солнечная система, где начало отсчета Солнце. Если рассматривали галактический диск, тогда бы центр галактики рассматривали.
Аноним 15/01/21 Птн 11:08:33 526569282
>>526568
Я не понимаю, зачем привязка к Солнцу или вообще звезде.
Потому что скорость тела можно отмерять относительно муравья, который находится в центре Солнечной системы.
При этом Массы то не сопоставимы как и влияние этих масс.
В уравнениях ничего не изменится в значения скоростей.
Аноним 15/01/21 Птн 11:26:37 526571283
>>526569
Попробую по-другому.
В системе из нескольких тел можно выбрать особенную систему отсчета, называемой центр масс. В этой системе уравнения движения становится очень просты. В случае гравирующих тел, относительно центр масс сохраняются секторные/угловые скорости. В другой произвольной системе отсчета уже появляется дополнительный член, который пидорасит все.
В Солнечной системе основная масса заключена в Солнце, поэтому его часто принимают за центр масс, пренебрегая малыми возмущения.
Суть гравитационного маневра в соблюдение двух условий: собственно сохранение импульсов + сохранение угловой скорости относительно центра масс.
Аноним 15/01/21 Птн 11:43:49 526573284
>>526571
>который пидорасит все.
Это потому что решение задачи трех тел в общем виде сложное? Или почему?
Аноним 15/01/21 Птн 11:55:02 526574285
>>526573
Не совсем.
Свойств центр масс геометрическое, короче это естественная симметрия уравнений движений. И оно актуально для систем любой сложности от двух тел до бесконечности. Симметрии очень удобный инструмент для нахождения или оценки решений. Даже задачу о двух телах решают через нахождении симметрий.
Аноним 15/01/21 Птн 12:06:24 526575286
>>526574
>Даже задачу о двух телах решают через нахождении симметрий.
Если это так просто, то почему задачу о трех не решают?
Аноним 15/01/21 Птн 12:08:06 526576287
D38CF285-4490-4[...].jpeg 180Кб, 1360x766
1360x766
Аноним 15/01/21 Птн 12:28:18 526578288
>>526575
Полный ответ не влезет в пост.
Кратко так:
У нас есть система дифференциальных уравнений. Прямо в лоб их не решить. Поэтому используют методы теории групп. Группа если что это множество/подмножество со определенной структурой.
Суть метода это нахождении симметрий, а те уже порождают группу, которое является множеством решений. И используя структуру на группе можно находить точные решения.
Однако среди групп есть очень пакостные, их еще называют неразрешимыми, которые задают структуру, которая никак не упрощается и поиск решений на них становиться еще сложнее. По этой же причине нельзя найти общего решения для уравнений выше 4-ой степени.
Впрочем методами теории групп можно найти частные решении для задачи трех тел, если есть в системе подходящие симметрии.
Аноним 15/01/21 Птн 12:35:59 526579289
Аноним 16/01/21 Суб 00:45:36 526607290
>>525425 (OP)
15 лвл, скоро 16
Ни разу не подтягиваюсь, хотя шестилетки на изи подтягиваються до 10 раз. Это они родились такими или они действительно тренятся? Просто сам пытался научится подтягиваться, 2 недели усердного процесса, 0 результата. Как?
Аноним 16/01/21 Суб 08:33:32 526613291
>>526607
Чтоб подтягиваться надо для начала научиться подтягиваться: просто напрячь мышцы в правильном порядке. Детям проще, у них короче рычаг и они быстрее учатся движениям.
Если у тебя проблемы, то найти любого турнкиеба, чтоб тебя научил путем поддержки/подстраховки.
Аноним 16/01/21 Суб 10:37:00 526617292
image.png 489Кб, 1366x768
1366x768
Думаю, это довольно известный таймплапс истории Земли с канала Algol на ютабе. Что означает эта полоска вдоль экватора?
Аноним 16/01/21 Суб 20:10:42 526646293
Господа подскажите что будет если сгенерировать ток очень высокой частоты скажем световой, и пустить его по проводу. Что будет? Будет ли светится провод, и пойдет ли такой ток вообще по проводу ведь это уже по сути свет
Аноним 16/01/21 Суб 20:18:36 526647294
>>526646
Что значит световой частоты? Любой переменный ток излучает "свет".
Аноним 16/01/21 Суб 20:18:54 526648295
>>526646
Не пойдет по проводу, металлы для частот видимого света непрозрачны, они отражают либо поглощают такое излучение. Для такого тока есть свои провода, отгадай название.
Аноним 16/01/21 Суб 20:19:27 526649296
>>526647
Он имеет ввиду поднять частоту переменного тока до частоты видимого света
Аноним 17/01/21 Вск 02:44:02 526656297
>>526649
Не совсем уверен что он это думает.
Думаю он не особо разбирается и думает что "света" там никакого из проводов на других частотах не выходит, если не видно глазами, и вообще раз не видно то жто не свет а вообще что-то другое.
Аноним 02/04/21 Птн 03:32:43 532322298
Что больше весит 1 кг пуха или 1 кг свинца?
Аноним 02/04/21 Птн 03:44:58 532323299
15307306284570.jpg 45Кб, 600x449
600x449
Аноним 02/04/21 Птн 04:17:01 532324300
Может ли дистиллированная вода быть радиоактивной?
Аноним 02/04/21 Птн 06:51:58 532326301
image.png 3422Кб, 1500x1497
1500x1497
>>532324
Если вместо протия - тритий, то да. Тритий радиоактивен.
Аноним 02/04/21 Птн 07:35:58 532327302
>>532322
Если взвешивать на поверхности Земли, то кусок свинца массой 1 кг будет весить больше, чем 1 кг пуха.
Аноним 02/04/21 Птн 09:18:25 532333303
image.png 580Кб, 1000x750
1000x750
Почему ихтиозавров считают рептилиями, а не амфибиями? Какие доказательства что ихтиозавры произошли отрептилий?
Аноним 02/04/21 Птн 17:38:34 532364304
>>525555
Давление меняется
Аноним 02/04/21 Птн 17:40:24 532365305
>>532364
Почему в самолёте меняется давление, если все крупные пассажирские самолёты герметичны и имеют систему поддержки давления?
Аноним 02/04/21 Птн 17:45:57 532366306
>>532365
Корпус самолета не абсолютно жесткий и при низком давление и температуре он распухает, создавая внутри пониженное давление.
Аноним 02/04/21 Птн 17:46:02 532367307
>>526044
>кому верить
В первую очередь не верить психолухам с их процентными измерениями усилий и ощущения превосходства
Аноним 02/04/21 Птн 17:53:49 532368308
>>532366
То есть система поддержания нормального давления просто не работает?
Аноним 02/04/21 Птн 18:08:57 532369309
>>532368
Работает, только у нее есть задержка. Скачки давления и температуры на высоте довольно сильными бывают.
Аноним 03/04/21 Суб 07:35:41 532394310
image.png 349Кб, 1229x897
1229x897
Где Китай с миллиардом умнейших людей на планете и топовой экономикой. Где Израиль? Почему немытая Россия обогнала Японию?
Аноним 03/04/21 Суб 07:45:47 532395311
Аноним 03/04/21 Суб 10:22:50 532414312
>>532365
В том то и дело что они имеют систему поддержки давления, а не герметичны. На высоте 11 км давление в салоне падает на четверть примерно. Если самолет делать полностью герметичным он будет слишком тяжёлым.
Аноним 03/04/21 Суб 10:28:08 532416313
>>532394
>Где Китай с миллиардом умнейших людей на планете
>умнейших людей на планете
)

>Где Израиль?
Самые умные жиды, Ашкеназы и Сефарды, в Израиле не живут
Аноним 03/04/21 Суб 10:32:23 532417314
>>532416
Неужели китайцы соснули?
Аноним 03/04/21 Суб 12:14:51 532423315
>>532417
А когда они не сосали?
Аноним 03/04/21 Суб 13:08:02 532424316
>>532423
Ну ты посмотри как поднялись, экономика которой завидуют США, бьют рекорды индустриализации, передовые технологии, может и наука подтянется
Аноним 03/04/21 Суб 13:57:51 532429317
>>532424
Сколько из этих технологий то китайцы придумали?
Аноним 03/04/21 Суб 14:13:27 532430318
16158363122580.png 627Кб, 1506x1388
1506x1388
>>532429
Народ жулик, народ копирка
Аноним 03/04/21 Суб 20:04:07 532454319
>>526607
>Ни разу не подтягиваюсь, хотя шестилетки на изи подтягиваються до 10 раз.
Чем меньше организм, тем легче им двигать. Муравьи вообще несколько своих весов в жвалах держат и помахивают. А слонам и бегемотам только для того, чтобы быстро ходить, надо адреналина ебануть, и спят они дохуя.
Аноним 03/04/21 Суб 20:22:30 532455320
>>532454
Объём и следовательно вес увеличивается в кубе, а сила мышц зависит от площади сечения, увеличивается в квадрате.
Аноним 03/04/21 Суб 20:49:42 532460321
>>526607
Либо ты кабанчик плотный тяжелый, либо долбоеб-аутист.
Аноним 04/04/21 Вск 10:03:46 532504322
Аноним 04/04/21 Вск 10:21:30 532508323
Аноним 04/04/21 Вск 12:04:13 532511324
15980240063500.jpg 213Кб, 927x674
927x674
Мне лень пихать телефон в резиновой перчатке под воду. Звук из динамика нормально распространяется под водой? На какое расстояние примерно? Если бы под водой жили роботы, они могли бы так же эффективно общаться речью, как люди на суше (в том же диапазоне частот)?
Аноним 04/04/21 Вск 15:07:16 532529325
Чем закон сохранения заряда отличается от закона сохранения барионного заряда и от закона сохранения лептонного заряда и зачем столько разных законов сохранения заряда?
Аноним 04/04/21 Вск 15:34:55 532532326
>>532529
Чтобы сохранялось
Аноним 04/04/21 Вск 17:43:33 532540327
Мои мысли по поводу путешествий во времени.
1. Путешествия в прошлое.
Любое, бесконечно малое изменение вселенной в прошлом повлечет за собой изменение измение картины мира в будущем, в том числе и в том моменте, когда вы отправляетесь или отправляете что-то в прошлое. А это логически невозможно. Путешествия в прошлое невозомжны.
2. Путешествия в будущее.
Если мы можем отправить что-то в будущее, значит это будущее определено. Потому как мы попадаем в уже определенный мир, а значит все, что было до этого, могло протекать только одним определенным образом. Следовательно, путешествия в будущее возможны только если мир детерминирован.
Я прав?
Аноним 04/04/21 Вск 18:06:39 532541328
>>532540
>Если мы можем отправить что-то в будущее, значит это будущее определено.
Нет не значит.
Аноним 04/04/21 Вск 18:23:30 532542329
>>532540
>Я прав?
Что такое время?
Аноним 04/04/21 Вск 19:31:35 532553330
>>532542
Время — форма протекания физических и психических процессов, условие возможности изменения[1]. Одно из основных понятий философии и физики, мера длительности существования всех объектов, характеристика последовательной смены их состояний в процессах и самих процессов, изменения и развития[2], а также одна из координат единого пространства-времени, представления о котором развиваются в теории относительности.

В философии — это необратимое течение (протекающее лишь в одном направлении — из прошлого, через настоящее в будущее)[3].

В метрологии — физическая величина, одна из семи основных величин Международной системы величин (англ. International System of Quantities, фр. International Système de grandeurs, ISQ)[4], а единица измерения времени «секунда» — одна из семи основных единиц в Международной системе единиц (СИ) (фр. Le Système International d’Unités, SI, англ. International System of Units, SI).
Аноним 04/04/21 Вск 19:41:20 532559331
>>532553
>и психических процессов
Вот ты и попался, в порашу в ph/ съеби.
Аноним 05/04/21 Пнд 03:10:33 532585332
Аноны, закон смещения вина

На самом деле распределение фотонов полностью равномерное в количестве ПО ЭНЕРГИИ, а смещение есть только потому, что на известном графике по оси икс длина волны-постоянная?

Т.е. смещение вина как бы есть, если смотреть по длине волны
Но если смотреть по энергии излучения то от пика в обе стороны всё равномерно?
Аноним 05/04/21 Пнд 03:26:51 532586333
Без названия.mp4 4996Кб, 400x216, 00:01:44
400x216
Аноним 05/04/21 Пнд 06:57:14 532594334
Аноним 05/04/21 Пнд 08:07:02 532599335
image.png 5Кб, 430x80
430x80
Аноним 05/04/21 Пнд 08:09:21 532600336
>>532529
>Чем закон сохранения заряда отличается от закона сохранения барионного заряда и от закона сохранения лептонного заряда и зачем столько разных законов сохранения заряда?

Это еще что! Есть еще законы сохранения странности и закон сохранения цвета. Физики наплодили сущностей и сами в них нихуя не разбираются.
Аноним 05/04/21 Пнд 09:20:02 532601337
>>532600
>Физики наплодили сущностей и сами в них нихуя не разбираются.
Смешно это слышать от долбоеба с обосцая.
Аноним 05/04/21 Пнд 10:49:02 532611338
>>532601
Ты хоть понимаешь о чем речь идет-то? В шкалке какие оценки по физике были?
Аноним 05/04/21 Пнд 13:18:41 532630339
>>532529
Ну прям ИСТИННЫЙ заряд можно назвать только электрический, ну еще можно добавить слабый изоспин( аналог заряда для слабого взаимодействия) и цветной заряд глюонов и кварков. Законы их сохранения следуют из симметрии полей, которые порождают заряд.
Остальные заряды не конкретно называть заряды(назвали зарядами по историческим причинам), лучше назвать числом барионов/лептонов, а законы сохранения их эмпирические и прямо не связаны с симметриями полей.
Аноним 05/04/21 Пнд 13:47:00 532635340
>>532630
>Ну прям ИСТИННЫЙ заряд можно назвать только электрический

Хочешь сказать, что закон сохранения лептонного заряда и закон сохранения барионного заряда ложные?
Аноним 05/04/21 Пнд 15:00:02 532640341
>>532611
>Ты хоть понимаешь о чем речь идет-то?
Если учесть, что про законы сохранения соответствующих зарядов рассказывают на любой приличной теоретической кафедре, то не понимать может только явный идиот вроде тебя.
Аноним 05/04/21 Пнд 15:47:16 532645342
>>532640
>не понимать может только явный идиот
Ну, раз ты не идиот и понимаешь, то расскажи что такое закон сохранения заряда, закон сохранения барионного заряда и закон сохранения лептонного заряда и почему нельзя обойтись одним законом сохранения заряда.
Аноним 05/04/21 Пнд 15:59:23 532648343
>>532635
Нет, они просто эмпирические и с ним не связаны симметрии полей. Короче они не являются фундаментальные величинами, прямо вытекающие из теории.
Ну еще во Вселенной наблюдается преобладание материи над антиматерии, что прямо указывает на нарушение этих законов.
Аноним 05/04/21 Пнд 16:29:28 532650344
>>532645
>то расскажи
Давать забесплатно лекции последнее что можно хотеть делать. Интересно, берешь любой учебник по ФЭЧ и вперед.
Аноним 05/04/21 Пнд 16:35:38 532651345
Феномен в физике это динамический процесс из частиц?
Аноним 05/04/21 Пнд 16:45:58 532654346
>>532648
>они просто эмпирические
Закон сохранения лептонного заряда и закон сохранения барионного заряда эмпирический? А закон сохранения заряда не эмпирический?
Т.е. закон сохранения лептонного заряда и закон сохранения барионного заряда можно как-то по-твоему наблюдать, а закон сохранения заряда наблюдать невозможно?

Я верно понял твои утверждения?
Аноним 05/04/21 Пнд 16:47:04 532655347
>>532650
Говорящая табуретка, плз.
Аноним 05/04/21 Пнд 17:14:02 532658348
>>532654
Нет, ты не понял не правильно.
Эмпирический закон строиться чисто по наблюдательным данным и у него нет теоретического обоснования. Короче по данным наблюдения увидели сильному закономерность и обозвали это законом.
Закон сохранения электрического заряда же имеет сильное теоретическое обоснование. Короче этот закон можно вывести из теории, что его роднит с другими фундаментальными законами вроде закона сохранения энергии и импульса.
Аноним 05/04/21 Пнд 17:20:54 532659349
>>532658
>у него нет теоретического обоснования
Так ты утверждаешь, что закон сохранения лептонного заряда и закон сохранения барионного заряда не имеют теоретического обоснования?


Аноним 05/04/21 Пнд 17:40:00 532660350
>>532659
ну типа того.
Конечно можно извратиться и кое как обосновать лептонное число через совокупность электрического заряда и слабого изоспина, но с барионное число вообще никак. Иначе бы не думали о распаде протона.
Аноним 05/04/21 Пнд 17:43:54 532661351
Аноним 05/04/21 Пнд 17:48:34 532662352
Аноним 06/04/21 Втр 03:40:37 532684353
Почему скорость электромагнитной волны в волноводе больше, чем в скорость света в вакууме?
Аноним 06/04/21 Втр 05:57:45 532686354
2019-01-1307-09[...].png 248Кб, 419x341
419x341
Аноним 06/04/21 Втр 08:37:39 532692355
Аноним 06/04/21 Втр 10:00:39 532696356
>>532662
>мы попадаем в уже определенный мир, а значит все, что было до этого, могло протекать только одним определенным образом.
Аноним 06/04/21 Втр 10:31:22 532698357
>>532696
>мы попадаем в уже определенный мир, а значит все, что было до этого, могло протекать только одним определенным образом.
Нет не попадаем, пока ты летишь в будущее у всех остальных время идёт нормально и процессы проходят непредсказуемо
Аноним 06/04/21 Втр 11:17:03 532699358
джеки-чан-мем-ш[...].jpg 29Кб, 552x360
552x360
Поясните за неинерциальную систему отсчёта. Почему она называется НЕинерциальной если в ней НАДО учитывать силы инерции чтобы все правильно подсчитать?
Аноним 06/04/21 Втр 11:23:19 532700359
Если выяснится, что физические константы изменяются со временем то как это повлияет на окружающий мир?
Аноним 06/04/21 Втр 11:35:09 532701360
>>532699
Перефразирую, силы инерции в неинерциальном поезде существуют потому что сам поезд движется по инерции, стало быть сама система обладает инерцией, то есть логично именно ее называть инерциальной
Аноним 06/04/21 Втр 13:13:40 532702361
продолжительнос[...].png 19Кб, 1010x272
1010x272
Как работает ожидаемая продолжительность жизни?
Если для мужиков при рождении она 68,2 года, а для шестидесятилетнего мужика - 16,8 лет, то выходит, что если сейчас родится мужик, ожидается, что он проживёт 68,2 года (и помрёт, соответственно в 2089 г.), а если мужику уже сейчас шестьдесят, что он доживёт до 76,8? Откуда такая разница? Почему новорожденный мужик должен прожить меньше, чем дед, родившийся 60 лет назад, в 1961 году, блять? Условия для жизни будут хуже в будущем, что ли?
Аноним 06/04/21 Втр 13:22:24 532703362
>>532699
>>532701
Инерция (от лат. inertia — покой, бездеятельность, постоянство, неизменность) — свойство тела оставаться в некоторых системах отсчёта в состоянии покоя или равномерного прямолинейного движения в отсутствие внешних воздействий.
Соответственно отсюда и название.
Аноним 06/04/21 Втр 13:23:00 532704363
>>532700
Никак. Потому что как известно, они не менялись на очень большем масштабе времени.
Аноним 06/04/21 Втр 13:31:48 532705364
>>532698
Ну, представь, что ты отослал предмет в будущее. А потом еще один предмет в то же будущее. Если все непредсказуемо, то первый предмет может попасть в мир, в котором была ядерная война, а второй в мир, в котором ничего не было. И что, эти предметы будут в разных мирах или что?
Аноним 06/04/21 Втр 13:37:10 532706365
>>532704
> на очень большем масштабе времени.
На каком именно?
Аноним 06/04/21 Втр 13:58:53 532707366
image.png 19Кб, 740x284
740x284
>>532705
Блять. Он не летит в другой мир. Они оба никуда не "попадут", а останутся в одном и том же мире и исход одного и того же случайного события для них будет одинаков, но не предопределён.
Аноним 06/04/21 Втр 14:54:36 532708367
>>532705
>Ну, представь, что ты отослал предмет в будущее.
На костер его!
Аноним 06/04/21 Втр 18:59:00 532715368
как это работае[...].mp4 12266Кб, 640x360, 00:04:10
640x360
>>525425 (OP)
1) ( см. прикрепленное видео )
Как это "работает" ?
Сколько времени будет светить и почему?

2) youtube.com/watch?v=2dajGoY9tpI
Что будет, если в таком опыте вместо человека будет курица?
Как все это делается ( в подробностях )?
Какие детали нужны?
Какие расчеты нужно провести, прежде чем такое собирать и делать?
Аноним 06/04/21 Втр 19:55:33 532720369
dna.jpg 49Кб, 487x557
487x557
Двач, почему цепи ДНК обычно антипараллельны? Пишут, что такая ориентация делает спираль стабильнее, но если так, то какие механизмы и принципы этого? Если мы сделаем 2 цепи ДНК параллельными, то, по крайней мере, водородные связи и стэкинг-взаимодействия сохранятся. Синтез все так же будет идти путем присоединения дНТФ к 3' концам, просто оба этих конца будут на одной стороне спирали. В чем плюсы антипараллельности, подскажите, пожалуйста.
Аноним 06/04/21 Втр 20:22:19 532721370
>>532715
Это работает только на территории Украины.
Аноним 06/04/21 Втр 21:01:47 532724371
>>532720
Собственно в стабильности + естественный отбор.
Нить ДНК благодаря дезоксирибозе ярко выраженную асимметрию относительно вращения и при связи с антипараллельной нитью имеет максимальный выигрыш энергии и следовательно такие структуры предпочтительнее.
У РНК такого нет.
Аноним 07/04/21 Срд 07:41:23 532734372
Как энергия может быть сжата вибрацией? Это прикол такой или реальность?
Аноним 07/04/21 Срд 09:28:49 532738373
>>532734
Ты о чём конкретно. Твоя фраза просто набор букв вне контекста.
Аноним 07/04/21 Срд 10:42:45 532743374
Правда ли мы на[...].mp4 12971Кб, 1280x720, 00:03:17
1280x720
Почему абсолютно все, даже биологи с мировым именем, говорят шимпанзе на 99% человек. Это же как "пи=3", не то что грубо, а фактически пиздежь. Когда людям откроют глаза?
Аноним 07/04/21 Срд 11:45:42 532744375
image.png 205Кб, 1024x388
1024x388
image2.png 215Кб, 1024x388
1024x388
>>532724

Спасибо за ответ. То есть симметрия, возникающая при антипараллельности цепей, позволяет иметь максимальный выигрыш энергии именно за счет оптимизации положения химических связей, т.е., из-за расположения, например, водородных связей в одной плоскости и т.д.?

Еще интересно, как праймаза узнает, в какую сторону ей двигаться? Она ведь всегда "читает" шаблонную цепь в направлении 3'-->5', хотя казалось бы, какая ей разница, она же все равно начинает синтез праймера de novo, могла бы ехать и в направлении 5'-->3', как на второй, неправильной картинке. Но такого не происходит. Получается, она эволюционно распознает направление сахарофосфатного остова цепи и может "ехать" только в направлении 3'-->5', потому что только в этом случае гибридная спираль (шаблонная+дочерняя цепь) будет антипараллельной?

Вирусные двуцепочечные РНК-спирали тоже антипараллельны.
Аноним 07/04/21 Срд 12:14:57 532747376
>>532743
>биологи с мировым именем, говорят шимпанзе на 99% человек
Пруф?
Аноним 07/04/21 Срд 12:22:42 532748377
>>532747
Например Северинов. Искать где он это говорит очень лень
Климатологи это что блять? Аноним 07/04/21 Срд 12:43:48 532750378
photo2021-04-07[...].jpg 215Кб, 720x1280
720x1280
Чё за пиздец с погодой происходит? На прошлых выходных было +30 в тени, сегодня снег идёт. Это из-за climate change?
Аноним 07/04/21 Срд 13:23:40 532755379
>>532744
Нет же. Одиночная нить ДНК сама по себе склонна скручиваться в винтовую линию, которая имеет выделенная направление - т.е нарушена симметрия. Сама по себе это не интересно, но важно когда две нити стыкуются и восстанавливают симметрию, что дает выигрыш энергии и повышенную стабильность. Впрочем вложить две параллельные винтовые линии без деформации одной из них невозможно. Может сам попробовать взяв две пружинки и сложив/скрутив их вместе.

>Получается, она эволюционно распознает направление сахарофосфатного остова цепи и может "ехать" только в направлении 3'-->5', потому что только в этом случае гибридная спираль (шаблонная+дочерняя цепь) будет антипараллельной?
Именно. ДНК-полимераза имеет строго расположенные в пространстве места, которые цепляются за фосфорные и гидроксильные группы ДНК и чувствительна к ее выделенному направлению. Зацепиться и идти по нити может только единственный образом.

РНК спирали не устройства, поскольку она сама не скручивается в винтовую линию, зато РНК может скручиваться произвольным образом, причем сама с собой.
Аноним 07/04/21 Срд 14:12:28 532756380
>>532734
Может, например берёшь пружину, берёшь болт, гайку и ударный шуруповёрт, и сжимаешь энергию вибрацией
Аноним 07/04/21 Срд 20:26:36 532785381
images.jpg 6Кб, 231x218
231x218
>>532755

Добра тебе. Теперь разобрался. Этот вопрос долго не давал мне покоя.
Аноним 08/04/21 Чтв 04:19:34 532834382
Аноним 08/04/21 Чтв 06:42:59 532840383
bcd5b01f53c878c[...].jpg 11Кб, 235x320
235x320
В Википедии намечается изменение политики, вышла статья про расизм, в частности там утверждается:
>The core beliefs uniting various types of racists are:
>That one group of people is inherently superior to another group of people.
>That white people are more intelligent than non-whites.
>That white people are more industrious than non-whites.
>That white people are more physically adept or attractive than non-whites.
>That white people are morally and ethically superior to non-whites.
>That the various cultures of white people are better than the cultures of non-white people.
>That white people have the right to live in a white-only nation.
>That a large majority of crimes are committed by non-whites.
Я согласен со всеми утверждениями - естественно, в определенных интерпретациях и контекстах, но всё равно довольно широких. Мне кажется, это не потому что я расист, а потому что таковы наблюдаемые факты. Т.е. это не вопрос идеологии, ценностей, угла зрения - есть тупо статистика, отрицать её это как отрицать сферичность Земли. Это не так? Или они там ебанутые все?
Что особенно смешно, в той же статье утверждается, что никаких "белых" нет, наука доказала.
Аноним 08/04/21 Чтв 08:44:17 532854384
>>532840
> наблюдаемые факты
Наблюдаемые факты это хорошо, но это только начало. Сами по себе нихуя не значат. К примеру вера в разного рода боженек это тоже результат наблюдаемых фактов. Помолился -> стало лучше -> исусик существует. Плоскоземельщики будут утверждать тебе что плоскость Земли соответствует наблюдаемым ими фактам, и наблюдаемые ими же факты свидетельствуют о фальсификациях с целью убедить окружающее быдло в сферичности Земли. На основе наблюдений можно только строить гипотезы (что конечно же охуенно, для первого шага). Для выводов нужен эксперимент. Это я не к тому что нигера какие-то умные или типа того. Я к тому что у тебя неправильный подход. Просто фактов (которые не являются результатами контроллируемого эксперимента) мало. Это уже не говоря о том что просто наблюдать мало - при сборе инфы важна методология. Просто случайные истинные факты могут ввести в заблуждение.
Аноним 08/04/21 Чтв 09:09:31 532855385
16146321668580.jpg 55Кб, 590x488
590x488
>>532756
В смысле болт продеть через пружину и закрутить шуруповертом? А причем тут вибрация? Сжимать же будет сам болт (нагрузка перейдет на его тело -> материал -> атомные связи).

>>532738
См пикрил
Аноним 08/04/21 Чтв 09:37:05 532858386
16178623419491.mp4 18395Кб, 640x352, 00:02:39
640x352
Аноним 08/04/21 Чтв 09:53:19 532859387
>>532858
Селедки пилят контент для быдла что бы заработать денег.
Аноним 08/04/21 Чтв 11:42:43 532869388
>>532840
Третье с конца полная хуйня, которая сама себе противоречит.

Статья про культурный феномен - расизм. Он оценивается негативно. Получается существуют "плохие" и "хорошие" культуры.
Аноним 08/04/21 Чтв 13:09:44 532881389
Если время это просто "четвёртое измерение", то почему оно измеряется в секундах, а не в метрах?
Аноним 08/04/21 Чтв 13:21:40 532884390
>>532881
>просто "четвёртое измерение"
>просто
А ты умеешь в операторы в гильбертовом пространстве?
Аноним 08/04/21 Чтв 13:37:26 532885391
>>532884
жидовские выдумки ненужны)
Аноним 08/04/21 Чтв 13:37:59 532886392
>>532884
А то! Я даже могу в симплектические формы.
Аноним 08/04/21 Чтв 13:46:27 532887393
>>532854
Ясно, раз контролируемого эксперимента поставить нельзя, то никакие факты о группах людей не являются значимыми. К примеру, несмотря на то, что рост среднего датчанина в среднем выше, мы не можем сказать, что датчане превосходят пигмеев по росту, а пигмеи превосходят датчан . Где эксперимент? Кстати, никаких датчан не существует, наука доказала.
Аноним 08/04/21 Чтв 13:47:08 532888394
>>532887
Пигмеи превосходят датчан в содержании подкожного меланина.
Аноним 08/04/21 Чтв 14:00:14 532893395
>>532888
>Пигмеи превосходят датчан в длине мпх
Аноним 08/04/21 Чтв 14:02:47 532896396
image.png 67Кб, 197x198
197x198
Аноним 08/04/21 Чтв 16:40:51 532901397
>>532884
Ну что-то такое слышал. Ты это к чему клонишь?
Аноним 08/04/21 Чтв 17:08:30 532903398
15939431792430.png 17Кб, 620x305
620x305
Аноним 08/04/21 Чтв 18:20:30 532907399
>>532881
Потому что простоватость-время завезли после того как запилили и обкатали механический формализм движения. Перепиливать его полностью не захотели вот отставили времени классическую размерность. Ну еще есть собственное время и для него нужна зарезервированная своя размерность. Иногда костыли нужно оставлять для принципа соответствия.
Аноним 08/04/21 Чтв 19:03:55 532912400
>>532881
Можно и в метрах. Оно там один хуй во все формулы входит как ct (c - скорость света).
Аноним 08/04/21 Чтв 19:14:08 532914401
Помогите люди добры. Я не понимаю. Почему так часто говорится о субъективности и наблюдении (измерении) в квантовой механике? Я спать нормально не буду. Если ИЗМЕРЕНИЯ и НАБЛЮДЕНИЯ заменить на ВЗАИМОДЕЙСТВИЯ, разве не встает все на свои места. Любое наблюдение - это взаимодействие и запутывание (в квантовом смысле). Но при чем тут СУБЪЕКТИВНОСТЬ? Почему запутывание не объективный процесс?
Кот Шреденгера был или жив или мертв в голове кого-то (субъективно!). Но в реальности он был во вполне конкретном состоянии - жив или мертв. То же и с фотоном, который запустил (или не запустил) аппарат по экзекуции кота. Фотон либо среагировал, либо нет. Он для всех без исключения либо среагировал либо нет. При чем тут субъективность? Неопределенность разве является субъективностью? Я не понмаю
Аноним 08/04/21 Чтв 20:59:58 532920402
>>532914
>Но в реальности он был во вполне конкретном состоянии - жив или мертв
В том и дело что нет.
>Он для всех без исключения либо среагировал либо нет.
Прочитай про эксперимент с квантовой бомбой.
Аноним 08/04/21 Чтв 22:36:25 532931403
>>532907
Почему тогда все три размерности пространства измеряются в метрах, а не отдельно по особой единице измерения на каждую размерность? Высота - в высотках, длина - в длинотках, ширина - в ширинотках, а время - в секундах. Было бы справедливо.

>>532912
Умножение на скорость меняет размерность же. Были секунды стали метры.
Аноним 08/04/21 Чтв 22:58:50 532935404
>>532914
Шредингер придумал научпоповское объяснение про кота для своей квантовой теории. На мой взгляд, оно лишь сбивает с толку. Квантовый мир контринтуитивен, там происходят чуждые нашему макромиру процессы. В микромире частица способна находится в разных состояниях и в разных местах одновременно. Представить это себе едва ли возможно, но похоже, что это именно так.

Про субъективность и объективность ничего не могу сказать.
Аноним 09/04/21 Птн 01:15:49 532945405
>>532920
>В том и дело что нет.
>>Он для всех без исключения либо среагировал либо нет.
>Прочитай про эксперимент с квантовой бомбой.
А сам ты в этой маняфантазии достаточно разобраться не смогу чтобы за неё пояснять?

Почему все квантовоманьки отправляют "загуглить"/"почитать про.." но ни один сам не показывает что он разобрался в том во что уверовал?
Аноним 09/04/21 Птн 01:22:55 532946406
Так всё-таки почему киловатт•час, а не киловатт/час?
Было бы как километр/час, количество мощности бы было просто киловатт, как расстояние километр, а "расход расстояния" километр/час

Хотя бля.. сейчас задумался... а почему километр/час, а не километр•час...
Аноним 09/04/21 Птн 03:12:35 532947407
>>532946
>Так всё-таки почему киловатт•час, а не киловатт/час?
Это условность, пищи так как тебя удобно, все поймут.
Аноним 09/04/21 Птн 03:14:26 532948408
>>532901
>Ты это к чему клонишь?
К тому, что это не "просто измерения". Вот ты можешь объяснить что такое пространство, что такое измерения и как оно все описывается?
Аноним 09/04/21 Птн 06:07:36 532962409
>>532947
Не, это же из формул идёт
Аноним 09/04/21 Птн 06:45:30 532965410
>>532947
Тащемта, киловатт•час - энергия/работа, а вот киловатт/час - производная от мощности, показывающая, как быстро она меняется. Чтобы показать, как быстро меняется энергия, нужен обычный (кило)ватт - КДж/с.
Аноним 09/04/21 Птн 07:48:44 532966411
>>532965
>Тащемта, киловатт•час - энергия/работа, а вот киловатт/час - производная от мощности

Что блядь?
Аноним 09/04/21 Птн 08:43:49 532967412
>>532945
Я ни один экзамен сдал по этой хуйне. А по квантовой бомбе.
Там довольно большое описание, а найти можно одним кликом в гугле.
>не показывает что он разобрался в том во что уверовал?
В смысле "показывает"? Как ты себе представляешь "показываение"?
Аноним 09/04/21 Птн 09:20:55 532968413
>>532948
> что такое пространство
Пространство это «только форма всех явлений внешних чувств»

Ну это если Канта раскурить. Хотя Кант вообще отделял время от пространства
Аноним 09/04/21 Птн 11:40:14 532971414
>>532946
> количество мощности бы было просто киловатт, как расстояние километр
Растояние это "количество выполненой работы" а не "количество мощности"
Тоесть у тебя соотвествуют:
"время" - общее для всех, часы
"мощность" - киловат и км/ч (скорость)
"количество выполненой работы" - киловатчас и км (растояние)

Несоотвествие в том что в разных случаях разные величины выражаются производными размерностями. В одном скорость ("мощность") выражается через растояние ("работу") и время. А во втором у тебя работа выражается через мощность и время.

Если ты выразишь скорость через непроизводную размерность (к примеру
узлы), то тогда растояние можно будет мерять в узлах•час прям как киловат•часы. Его собственно так и меряли/меряют. Только используют другое название: узел•час это морская миля*.


Аноним 09/04/21 Птн 12:22:07 532973415
>>532968
>Пространство это «только форма всех явлений
Что такое "форма"?
Аноним 09/04/21 Птн 12:24:26 532976416
Аноним 09/04/21 Птн 12:42:48 532977417
image.png 1137Кб, 853x480
853x480
image.png 964Кб, 450x698
450x698
>>532840
>more intelligent
Средний IQ азиатов и евреев выше.

>more industrious
Большая часть индустрии в Азии в 21 веке.

> more physically adept
Только почему-то начиная с фашистской олимпиады "арийцы" сосут у негров.

>attractive
Мммаксимум не объективно. Вообще никак не измеряется, ни в каких единицах.

>morally and ethically superior
Настало время напомнить, что концлагери смерти, атомную бомбу и легализацию гей-браков придумали белые.

>cultures of white people are better
Ничем не измеряется. Не объективно. А субъективно культура светлых европейцев и светлых американцев выглядит очень однообразной и скудной в сравнении культурами Индии и Китая.

>white people have the right to live in a white-only nation
Ровно на столько, на сколько они способны реализовать это право. Пока, судя по всему, не способны никак.

>large majority of crimes are committed by non-whites
Пожалуй единственный аргумент, который пока подтверждается.
Аноним 09/04/21 Птн 12:55:12 532979418
>>532977
>Средний IQ азиатов и евреев выше.
Так он же про интеллект говорит, а не абстрактную циферку, которая измеряет хуй пойми что.

>Большая часть индустрии в Азии в 21 веке.
Кто является владельцем индустрии?

>Только почему-то начиная с фашистской олимпиады "арийцы" сосут у негров.
Олимпиада это соревнование мутантов на фарме.

>Мммаксимум не объективно. Вообще никак не измеряется, ни в каких единицах.
Интеллект у тебя можно измерить одной циферкой, а привлекательность нельзя?

>Настало время напомнить, что концлагери смерти,
Рабовладельческие империи в Африке можно считать концлагерями смерти или нынче это пионерские лагеря?

>атомную бомбу и
Которая обеспечила мир.

>легализацию гей-браков придумали белые.
Пидорство это болезнь, которая не относится даже косвенно к этносу и культуре.

>Ничем не измеряется.
См. пункт про IQ.

>в сравнении культурами Индии и Китая
Которые проиграли соревнование с культурой европейцев. К тому же китайцы почти всю историю были под цензурой.

>Ровно на столько, на сколько они способны реализовать это право.
Справедливо.
Аноним 09/04/21 Птн 14:13:52 532983419
Энергия связи и, соответственно, дефект массы - результат взаимодействия глюонов? Т.е., они "забирают" на себя эту энергию, а в данном случае - массу нуклонов, а при расщеплении ядра на составные части "отдают обратно"?
Аноним 09/04/21 Птн 14:18:23 532984420
>>532966
Ватт = 1 Дж / с
Ватт/секунда = Дж/с2
Киловатт/час показывает, насколько быстро растёт мощность. А энергия, таким образом, растёт в геометрической прогрессии.
Киловатт/час != Киловатт*час
Аноним 09/04/21 Птн 14:21:00 532985421
>>532912
Вполне логично, кстати. Говоришь, что между событиями на временной шкале расстояние в 3x109 м - получается, что между ними прошло 10 секунд.
Аноним 09/04/21 Птн 22:19:19 533025422
>>532983
Очень грубо говоря, да. Есть дохуя тонкостей и подводных как частицы/кванты полей обретают наблюдаемую массу.
Дефект масс возникает из-за того, что в потенциальной яме частицы обладают отрицательной энергией. И конечная полная энергия системы оказывается меньше, чем сумма ее составляющих по отдельности. Потенциальная создается чисто полевым методом и тут уже лезут много тонкостей.
Для ядер потенциальную создает взаимодействие Юкавы, производное от сильного взаимодействие и внезапно электрослабого. С массой нуклонов еще больше тонкостей, ибо там большая часть массы создается массовой щелью сильного взаимодействия, а с ней дохуя проблем.
09/04/21 Птн 23:05:58 533027423
Поясните за плоскопараллельное движение, написано что это
>вид движения абсолютно твёрдого тела, при котором траектории всех точек тела располагаются в плоскостях, параллельных заданной плоскости.
И тут же
>Примером плоскопараллельного движения по отношению к вертикальной плоскости, относительно которой тело движется в параллельном направлении, является качение колеса по горизонтальной дороге.
Схуяли? Возьмем точку на ободке колеса, как только колесо совершит оборот, точка опишет дугу, плоскость образуемая этой дугой будет пересекать плоскость дороги. а по условию должна быть параллельна заданной плоскости!
Аноним 09/04/21 Птн 23:21:27 533028424
Аноним 10/04/21 Суб 08:18:24 533037425
>>532979
>>Средний IQ азиатов и евреев выше.
>Так он же про интеллект говорит, а не абстрактную циферку, которая измеряет хуй пойми что.
Ну давай альтернативные способы измерения. Ах! Ты не можешь их дать, потому что их нет))

>>Большая часть индустрии в Азии в 21 веке.
>Кто является владельцем индустрии?
Ты так спрашиваешь, будто у индустрии могут быть какие-то определённые владельцы. Бессмысленный вопрос.


>>Только почему-то начиная с фашистской олимпиады "арийцы" сосут у негров.
>Олимпиада это соревнование мутантов на фарме.
Спортивную фарму изобрели после войны. Фашистская олимпиада была до войны. Видишь противоречие?

>>Мммаксимум не объективно. Вообще никак не измеряется, ни в каких единицах.
>Интеллект у тебя можно измерить одной циферкой, а привлекательность нельзя?
Ну давай покажи объективные методы измерения привлекательности.

>>Настало время напомнить, что концлагери смерти,
>Рабовладельческие империи в Африке можно считать концлагерями смерти или нынче это пионерские лагеря?
Продавали рабов белым людам. Живых рабов. А белые люди делали живых рабов мёртвыми.

>>атомную бомбу и
>Которая обеспечила мир.
Да. И покой. Вечный. Множеству ни в чём не повинных людей.

>>в сравнении культурами Индии и Китая
>Которые проиграли соревнование с культурой европейцев. К тому же китайцы почти всю историю были под цензурой.
Нельзя проиграть в соревновании, если ты не участвуешь в соревновании. Они не проиграли.

Аноним 10/04/21 Суб 11:50:50 533048426
Аноним 10/04/21 Суб 12:30:59 533051427
>>533037
>Ну давай альтернативные способы измерения.
Я не утверждал, что они у меня есть. Я всего лишь сказал, что твоя непонятная циферка вообще непонятно что измеряет и откуда взялась.

>будто у индустрии могут быть какие-то определённые владельцы
Владельцы частной собственности из одного экономического сектора. Это владельцы индустрии.

>Видишь противоречие?
Противоречий нет никаких, твоя олимпиада не показатель. Ну если мы не измеряем показатели медицинских препаратов.

>Ну давай покажи объективные методы измерения привлекательности.
Ты на вопрос не ответил.

>Продавали рабов белым людам. Живых рабов. А белые люди делали живых рабов мёртвыми.
Белые покупали рабов, чтобы убить? Интересно. Ты тролль?

>Да. И покой. Вечный. Множеству ни в чём не повинных людей.
Этих жертв можно было легко избежать. Виновата не бомба, а люди.

>Нельзя проиграть в соревновании, если ты не участвуешь в соревновании. Они не проиграли.
Колониализм и опиумные войны говорят о том, что проиграли. О твоем участии в соревновании никто не спрашивает, у нас тут нет комитета, который это контролирует и берут всех. Хотят они того или нет. Вряд ли всякие вымершие виды об этом просили.
Аноним 10/04/21 Суб 14:48:22 533059428
Как представить что когда то не было абсолютно ничего, а потом все спонтанно появилось? Или наоборот, что некая реальность существовала всегда и её просто нужно принять как данность? Подозреваю что это как-то связано с восприятием и природой времени и возможно стоило пойти с этим в /пх но вдруг у научников есть что сказать на этот счёт.
Аноним 10/04/21 Суб 15:18:17 533063429
Аноним 10/04/21 Суб 15:18:57 533064430
>>533027
> по условию должна быть параллельна заданной плоскости
Ну так у тебя в примере задана вертикальная плоскость (" по отношению к вертикальной плоскости"), а не плоскость горизонтальной дороги.
Аноним 10/04/21 Суб 15:32:31 533066431
>>533059
>Как представить что когда то не было абсолютно ничего, а потом все спонтанно появилось?

>Или наоборот, что некая реальность существовала всегда и её просто нужно принять как данность?

>>533064
Спасибо, точно!
Аноним 10/04/21 Суб 20:34:44 533076432
>>533059
Философия этим уже особо не занимается; с века, этак, двадцатого метафизика не в почёте. А если найдутся какие-то труды - то их лучше применять как наглядная иллюстрация стимуляторного психоза.
Аноним 10/04/21 Суб 23:00:55 533080433
Когда говорят, что ничего не может двигаться со скоростью больше скорости света, то имеют в виду относительно чего? Два фотона, которые двигаются навстречу, имеют скорость друг относительно друга 2х скорости света.
Аноним 11/04/21 Вск 00:11:55 533085434
Аноним 11/04/21 Вск 02:13:37 533089435
>>533080
>Два фотона, которые двигаются навстречу, имеют скорость друг относительно друга 2х скорости света.
>>533085
А относительно стороннего наблюдателя?

Наблюдатель смотрит на отрезок расстояния 1С
С одного и второго края его навстречу друг-другу вылетают фотоны
Фотоны друг+относительно друга движутся со скоростью 1С
Но для стороннего наблюдателя пройдут участок за время 1С/2С
Аноним 11/04/21 Вск 02:36:57 533091436
>>533085
>>533089
Какой же у вас уебищный тред. Название литералли тред тупых вопросов, сюда люди приходят спросить что им непонятно, а в результате жалкие чмошники вместо ответа срут в тред.
И это блядь не первый раз, я уже задавал вопрос, на который два дня мне срали, что толсто, а потом оказалось что я изначально был прав.
Аноним 11/04/21 Вск 02:47:48 533092437
>>533091
>а потом оказалось что я изначально был прав
Это лишь значит, что ты остался таким же тупым и просто не понял даже ответа. Есть особенный уровень вопросов, из-за которых проще послать нахуй. Это либо платиновые вопросы как про два фотона, либо настолько тупорылые, что сразу понятно, что у автора вопроса интеллект табуретки и он не в состоянии воспринимать ответ, либо вопросы, где сразу видно, что спрашивающий имеет своё "авторитетное" мнение по этому поводу и просто хочет начать вонять в треде. Как например последнее время тут посилившийся шизик, пытающийся с образованием 9-ти классов, придумать собственную интерпретацию квантовой механики.
Аноним 11/04/21 Вск 02:54:27 533093438
>>533092
Мой тогдашний вопрос был не про квантовую механику, а по МТДТ. Два дня школьники с синдромом вахтера итт срали, пока не пришёл какой-то случайный анончик и таки ответил по теме, обосрав всех местных долбоебов знатоков.
С фотоном будет аналогично.

>>533080
Бамп вопросу.
Аноним 11/04/21 Вск 03:08:16 533094439
>>533093
>Бамп вопросу.
Какому вопросу, даунич?
Там утверждение.

Какого ответа ты ждёшь на утверждение?
Ответное утверждение что такие утверждения делают долбаёбы?
Аноним 11/04/21 Вск 03:12:28 533096440
>>533094
Дегенерат, ты собрался тут ещё и клоуном подработать в дополнение к вахтерству? Пшел нахуй, в посте есть вопрос.

>>533080
Бамп вопросу.
Аноним 11/04/21 Вск 03:16:56 533097441
>>533096
Дурачка тупая, почему ты порвалась...

Так зачем ты к вопросу прикрепил утверждение?
Аноним 11/04/21 Вск 04:40:47 533099442
>>533093
>С фотоном будет аналогично.
Это платиновый вопрос. На него даже как-то неприлично отвечать. Это как отвечать на фразочки "скажи триста".
Аноним 11/04/21 Вск 05:52:02 533102443
Какого цвета нейтроний? При условии, что он остывший конечно.
Аноним 11/04/21 Вск 07:00:15 533103444
>>533080
С фотонами всё сложно. В системе координат фотона время остановилось и фотон не движется никак, а вместо движения находится одновременно во всех точках своего пути. Во всех точках своего пути фотон возник мгновенно и одновременно. То есть фотон относительно фотона не перемещается, а сразу существует одновременно везде.
Аноним 11/04/21 Вск 07:02:09 533104445
Аноним 11/04/21 Вск 08:02:19 533108446
>>533103
Два фотона, которые двигаются навстречу, имеют скорость друг относительно друга 2х скорости света.
Аноним 11/04/21 Вск 08:11:33 533109447
Аноним 11/04/21 Вск 08:18:06 533110448
>>532967
>В смысле "показывает"? Как ты себе представляешь "показываение"?
Кратким описанием сути, но таким, чтобы оно не было тпшны "ни о чём"
Аноним 11/04/21 Вск 08:37:17 533111449
>>533109
Но у него есть некая логика. Если 2 машины едут навстречу друг другу со скоростью 60 км/ч, то расстояние между ними сокращается со скоростью 120 км/ч. Так ведь?
Аноним 11/04/21 Вск 08:43:27 533112450
>>533111
>то расстояние между ними сокращается со скоростью
Для кого?
Аноним 11/04/21 Вск 08:47:27 533113451
>>533112
Для мамки твоей)
Хуй соси)))
Аноним 11/04/21 Вск 10:46:45 533120452
>>533110
Ты же понимаешь, что научный эксперимент, он описывается обычно максимально компактным и концентрированным образом. Краткое описание, это и есть сама научная статья об этом эксперименте.
Но если совсем кратко, это решение задачки "у тебя есть бомба, детенатор у которой срабатывает от одного фотона. Разбор бомбы тоже её взорвет. Как среди таких бомб найти работающую, но при этом не взорвать её?"
Аноним 11/04/21 Вск 10:58:11 533121453
>>533113
Ой, ты почему так порвалась дурочка тупая...
Аноним 11/04/21 Вск 11:08:18 533122454
image.png 1Кб, 137x63
137x63
>>533108
>>533111
>Если 2 машины едут навстречу друг другу со скоростью 60 км/ч, то расстояние между ними сокращается со скоростью 120 км/ч. Так ведь?
Так, но на скоростях близких к скорости света не так.
Аноним 11/04/21 Вск 12:23:54 533124455
Какую константу надо поменять, чтобы во вселенной наступил полный пиздец?
Аноним 11/04/21 Вск 12:34:24 533126456
>>533124
Надо сначала дать определение полному пиздецу.
Аноним 11/04/21 Вск 13:00:39 533127457
>>533124
Гугли "антропный принцип". Там дохуя чего можно поменять так чтобы все наебнулось.
Аноним 11/04/21 Вск 14:00:50 533128458
>>533080
Бамп вопросу.
>Когда говорят, что ничего не может двигаться со скоростью больше скорости света, то имеют в виду относительно чего?
>Два корабля, которые двигаются навстречу, имеют скорость друг относительно друга 2х скорости света.
Аноним 11/04/21 Вск 14:16:30 533129459
>>533128
> оторые двигаются навстречу, имеют скорость друг относительно друга 2х скорости
Нет. Тебе же уже ответили. Даже формулу правильную вкинули, вместо той хуйни по которой ты 2х высчитал. Будет не больше скорости света.
Аноним 11/04/21 Вск 14:27:45 533130460
>>533129
Никто не ответил на мой единственный вопрос в треде, вы там срались по поводу фотонов и движения навстречу. Вопрос был не про это.

>>533080
Бамп вопросу.
Аноним 11/04/21 Вск 14:31:48 533131461
>>533130
Мудила гороховая, тебе уже ответили на твой вопрос. При движении на таких скоростях скорость складывается через хитровыебаную формулу, по которой относительная скорость двух объектов летящих на встречу друг другу со скоростью света будет равна скорости света.
Аноним 11/04/21 Вск 14:32:52 533132462
>>533130
Относительно любой инерциальной системы отсчета движение не может происходить выше скорости света.
Аноним 11/04/21 Вск 14:38:13 533133463
>>533128
Относительно себя мб?
Аноним 11/04/21 Вск 14:39:13 533134464
>>533131
Дегенерат, никто не ответил мой вопрос до него >>533132
Вот первый ответ по теме за сутки, как я и говорил придет адекват и ответит, пока школьники вахтеры итт будут корчиться в попытках не отвечать на тупые вопросы в треде тупых вопросов.

>>533132
Окей, а вселенная расширяется с какой скоростью? Если считать относительно места, где произошёл взрыв, она расширяется во все стороны и с каждой стороны со скоростью света, разве нет?
Аноним 11/04/21 Вск 14:39:18 533135465
>>533131
Но это ломает логику встречного движения.
Аноним 11/04/21 Вск 14:39:39 533136466
>>533080
>Когда говорят, что ничего не может двигаться со скоростью больше скорости света

Ты видать не так понял. Двигаться со скорость больше скорости свта ничто не запрещает, нельзя передавать информацию со скоростью больше скорости света.
Аноним 11/04/21 Вск 14:41:13 533137467
>>533134
Пространство может расширяться быстрее ящитаю.
мимо
Аноним 11/04/21 Вск 14:43:32 533138468
>>533135
Это ломает твое представление о логике встречного движения
Аноним 11/04/21 Вск 14:45:32 533139469
>>533134
> Если считать относительно места, где произошёл взрыв, она расширяется во все стороны
Расширение вселенной никак не связано с движением. Меняется метрика пространства. У Вселенной нет центра. Если ты станешь в любой точке Вселенной то тебе будет казаться что все отдаляется от тебя (тоесть что ты стоишь в центре).
>>533135
Только выдуманую тобой логику.
Аноним 11/04/21 Вск 14:45:54 533140470
>>533134
Ты понимаешь что ты поехавший? Тебе отвечают на твой дегенеративный вопрос уже второй или третий раз, а ты под дурачком прикидываешься и игнорируешь ответы.

>Если считать относительно места, где произошёл взрыв
Нет такого места
Аноним 11/04/21 Вск 14:53:34 533141471
>>533140
Не гони, годный у него вопрос.
Аноним 11/04/21 Вск 15:15:12 533142472
1618143309433.png 28Кб, 631x1000
631x1000
>>533122
Тут одно из двух - либо к формуле шли важные пояснения, которые ты забыл приложить, либо формулу подгоняли под реальность как могли и она не выводится из законов.
Проверить просто. Посчитай мне vrel с условиями на моей пикче и покажи для нее
>инерциальную систему отсчета
Аноним 11/04/21 Вск 15:16:07 533143473
>>533142
Алсо красное тело тоже летит по орбите, просто она настолько большая, что кривизну можно проигнорировать
Аноним 11/04/21 Вск 16:30:41 533145474
>>533142
>мало/многолетний дебил не осилил векторное исчисление
Ну тут всё ясно.
Аноним 11/04/21 Вск 16:39:23 533147475
Аноним 11/04/21 Вск 16:43:03 533148476
Аноним 11/04/21 Вск 22:01:35 533159477
Можно ли вместо пены для бритья использовать свежие фекалии?
Аноним 11/04/21 Вск 22:35:42 533162478
>>533159
Смотря для какой цели. но в общем да можно. Почему бы и нет?
Аноним 12/04/21 Пнд 06:24:39 533168479
Аноним 12/04/21 Пнд 08:50:14 533175480
>>533159
Если заражение крови не пугает то можно
Аноним 12/04/21 Пнд 12:06:01 533184481
>>533124
Как будто эта вселенная сама не полный пиздец.
Аноним 12/04/21 Пнд 13:24:22 533186482
Аноним 12/04/21 Пнд 16:26:27 533192483
>>525425 (OP)
Можно ли ускорить эволюцию человека с помощью тотальной евгеники?
Аноним 12/04/21 Пнд 16:37:25 533193484
>>533192
Нет. Евгеника мистическая маня-хуита.
Ускоряется эволюция только за счет быстрой сменой поколений и быстрыми мутациями, что дает много побочек. В добавок помимо генов есть еще их экспрессия, которые значительно усложняет отбор признаков.
Аноним 12/04/21 Пнд 17:05:20 533196485
>>533193
Каким образом применение селекции к человеку это "мистическая маня-хуита"?
Аноним 12/04/21 Пнд 17:19:30 533199486
>>533196
Все дело в методике отбора, он чисто мистический. Экспрессия генов имеет такое хитросплетение, что отбором по одному признаку придешь к совершенному другому что хотел.
Если уж улучшать людей, то через прямо генную инженерию и эпигенетику.
Аноним 12/04/21 Пнд 17:55:36 533202487
>>532715
bump моему вопросу
Аноним 12/04/21 Пнд 18:23:42 533204488
>>532715
> Как это "работает" ?
Какая-то батарейка или типа того. Там на видео очевидный монтаж - когда он переключает штепсель. Они монтировали ориентируясь на руку - поэтому она движется более-менее нормально сначала вверх потом вниз. Но они забыли (либо скорее всего просто не захотели ебаться - видео очевидным образом расчитано на каких-то легковерных дурачков) про то что еще нужен перенос штепселя. Тоесть должно быть движение руки вверх - перенос - движение руки вниз. А у них штепсель сразу телепортируется с положения над одним удлинителем к положению над вторым. Что указывает на крайне низкий уровень контента - подобного поведения можно добиться элементарными модификациями удлинителя, тоесть никакой монтаж не нужен.
Аноним 12/04/21 Пнд 20:07:08 533218489
>>533199
>Все дело в методике отбора, он чисто мистический.
Как тогда вышло, что породы собак отобрать сумели и они работают? Бойцовые - бойцуют, сторожевые - сторожат, пудели - считают палочки в цирке.
Аноним 12/04/21 Пнд 20:35:44 533220490
>>533218
Сравнил, лол.
Большинство пород собак выведено случайно и вообще породы стали породами постфактум. Вывести селекций намерено мопса или питбуля из волка практически невозможно, проще лотерею с первого раза выиграть.
Ну еще породы выделяются строго определенными признаками, при этом другие признаки в расчет не берут. С человек будет так же. Захочешь вывести человека с метровым хуйцом - получить тупого быдлами с мозгом хлебушком. Захочешь вывести сверхгения с 400 асикью - получишь задохлика аутиста с букетом уродств.
Аноним 12/04/21 Пнд 20:37:52 533221491
>>533220
>Захочешь вывести сверхгения с 400 асикью - получишь задохлика аутиста с букетом уродств.
Похоже на рыдания "вриооти!!1" сжв- новиопа какого-то
Аноним 12/04/21 Пнд 22:07:16 533234492
>>533220
>Захочешь вывести человека с метровым хуйцом - получить тупого быдлами с мозгом хлебушком. Захочешь вывести сверхгения с 400 асикью - получишь задохлика аутиста с букетом уродств.
Ну ИРЛ так всё и происходит. Ты разве не чувствуешь себя продуктом селекции безвольных, послушных рабов на протяжении последних лет трёхсот?
Аноним 12/04/21 Пнд 22:24:58 533239493
>>533221
Ты глупенкий или что? Какое отношение породы вообще имеют к эволюции? Породы собак это просто отобраные в соотвествие с описанием породы особи. Порода поддерживается с помощью того что весь не соответсвующий описанию приплод забивают. Как только человек прекратит свои усилия по выводу пород - все породы исчезнут. Лет через 100 будут бегать одни волкоподбные дворняги. Волк - результат эволюции. Породы собак - селекционная хуйня котороя существует только благодоря въебуемым человеком в это ресурсов. Также и твоя эвгеника - максимум что у тебя будет это зоопарк фриков. Который накроется как только закончится бабло. Если ты хочешь целенаправленно ускорять эволюцию то это только генная инженерия.
Аноним 13/04/21 Втр 15:32:31 533283494
>>533239
>максимум что у тебя будет это зоопарк фриков
Новиоп скозал?
Аноним 14/04/21 Срд 15:00:14 533389495
Аноним 14/04/21 Срд 15:01:18 533391496
>>533389
Потому что нет никаких границ между неживым и живым. Это чисто религиозная дихотомия.
Аноним 14/04/21 Срд 15:50:45 533394497
>>533389
>ведет себя как живое?
Это что вообще значит? У него там метаболизм происходит? Может оно размножается? Или "ведет себя как живое" заключается в том что оно движется?
Аноним 15/04/21 Чтв 02:01:18 533424498
>>533391
> чисто религиозная дихотомия
Чем "научные" дихотомии лучше? Почему на "плоскости" больше "точек", чем на прямой "линии"?
Аноним 15/04/21 Чтв 07:51:15 533430499
>>533424
Мощности множеств разные.
Аноним 15/04/21 Чтв 11:36:37 533441500
перекат.gif 4140Кб, 444x250
444x250
Аноним 15/04/21 Чтв 19:31:45 533470501
>>533424
>Чем "научные" дихотомии лучше
Тем, что не претендуют на истину в последней инстанции. У тебя есть геометрия, где параллельные прямые не пересекаются, а есть такая, где наоборот пересекаются. Пользуйся любой на здоровье, обе вполне себе живые и даже на практике обоими пользоваться можно. А большая часть споров на этой доске споры о семантике, нежели о сути явлений, что довольно уныло.
Аноним 17/04/21 Суб 05:58:19 533551502
Аноним 12/05/21 Срд 18:51:02 535178503
1
Настройки X
Ответить в тред X
15000
Макс объем: 40Mб, макс кол-во файлов: 4
Кликни/брось файл/ctrl-v
Стикеры X
Избранное / Топ тредов